Будь умным!


У вас вопросы?
У нас ответы:) SamZan.net

Эта работа требует не только большого упорства но и умения без которого затрата сил и времени не дает долж

Работа добавлена на сайт samzan.net:


ОБЩИЕ МЕТОДИЧЕСКИЕ УКАЗАНИЯ

Данное пособие ставит своей целью оказание помощи студентам в организации их самостоятельной работы по овладению системой знаний, умений и навыков в объеме действующей  учебной программы.

Эта работа требует не только большого упорства, но и умения, без которого затрата сил и времени не дает должного эффекта. Читать, понимать прочитанное и применять его практически — вот в чем суть умения работать с учебными пособиями.

Некоторые практические советы. 

Прежде всего, необходимо ознакомиться с содержанием программы. Затем следует выбрать в качестве основного учебное пособие и придерживаться его при изучении всей части курса, так как замена учебника может привести к утрате логической связи между отдельными вопросами.

Конспекты по математике главным образом должны содержать определения, чертежи и выводы основных формул. Записи должны быть аккуратными. Не нужно забывать, что они делаются для того, чтобы впоследствии ими пользоваться.

Учитесь самоконтролю. Для студента это важнейшая форма проверки правильности понимания и усвоения материала.

Помните: учебник нужно не просто читать, а изучать; основы запоминания является понимание, знание забывается — понимание никогда; повторение — важнейшее средство, предотвращающее забывание; необходимо выработать привычку систематической самостоятельной работы, «натаскивание» к экзамену дает слабые н поверхностные знания.

О решении задач. Решение задач является лучшим способом закрепления материала. Конечно, общих рецептов для решения разнообразных задач не существует, рекомендуем придерживаться следующих советов:

1 Величины, данные в условии задачи, необходимо одну систему единиц; нарушение этого правила. является распространенным источником ошибок у учащихся.

2. Внимательно изучите цель, поставленную в задаче; выявите, какие теоретические положения связаны с данной задачей в целом или с некоторыми ее элементами.

3. Не следует приступать к решению задачи, не обдумав условия и не найдя плана решения.

4. Попытайтесь соотнести данную задачу к какому-либо типу задач, способ решения которых вам известен.

5. Если не видно сразу хода решения, то последовательно отвечайте на вопросы: что дано; что нужно найти; достаточно ли данных, чтобы найти неизвестное, и т. п.

6. Попробуйте расчленить данную задачу на серию вспомогательных, последовательное решение которых может составить решение данной задачи.

7. Найдя план решения, выполните его, убедитесь в необходимости и правильности каждого шага, произведите проверку решения и, если нужно, его исследование.

8. Подумайте, нельзя ли было решить задачу иначе; известно, что одна и та же задача может иметь несколько решений, поэтому следует выделить наиболее рациональное,

9. Если решить задачу не удается, следует отыскать в учебной (или популярной) литературе уже решенную задачу, аналогичную ей, (похожую на данную), изучите внимательно это «готовое» решение и постарайтесь извлечь из него пользу для решения своей задачи.

Контрольные работы следует выполнять самостоятельно и лишь после того, как проработан соответствующий теоретический материал и решен необходимый минимум задач. Так как каждой теме соответствует задача или упражнение, то контрольную работу следует выполнять постепенно по мере изучения материала.

При решении задач следует обосновать каждый шаг решения, исходя из теоретических основ курса. Не следует применять формулы, которые не входят в  учебную программу. Решение должно быть доведено до окончательного ответа.

10. Изучая материал, составьте конспект, содержащий определения, чертежи, вывод основных формул. Записи должны быть аккуратными, чтобы в дальнейшем можно было без труда  им воспользоваться. Лучшим способом закрепления материала является решение задач. Если ход решения задания не ясен, то последовательно отвечайте на вопросы: что дано, что  нужно найти, достаточно ли данных, чтобы найти неизвестное и т.д.

Обратите внимание на оформление контрольной работы:

1.Контрольную работу необходимо выполнить на листах формата А-4 с полями для замечаний преподавателя.

2.На титульном листе (1-2) утвержденного образца аккуратно и разборчиво следует заполнить все его данные.

3. Для записей необходимо использовать пасту  темного цвета , кроме красного.

4. Условие задания следует записать полностью.

5. Решение задач  следует сопровождать  краткими пояснениями, записывая используемые в ходе решения формулы.

6.  Чертежи следует выполнять  карандашом, используя чертежные инструменты.

7. Каждое задание следует выполнять с новой страницы.

8. В конце работы указывается  используемая  литература, ставится дата и личная подпись.

9. Если студент допустил в работе недочеты, ошибки, то он должен выполнить все указания преподавателя, сделанные в рецензии.

10. Если студент выполнил работу не по своему варианту, то работа возвращается ему без оценки.

11. Студенты, не получившие зачета по контрольной работе, к экзамену не допускается.

12. Зачтенные контрольные работы студент сохраняет и обязательно предъявляет преподавателю при сдаче  экзамена. Студент  выполняет по высшей математике две контрольные работы №1(содержит 5заданий) в первом полугодии и №2(содержит 6заданий) во втором полугодии.

Номер контрольных заданий определяется по зачётной книжке.

ЛИТЕРАТУРА

Основная

1. Алгебра и начало анализа/Под ред. Г. Н. Яковлева.М., 1981,ч. I.

2. Алгебра и начала анализа/ Под ред. Г.Н. Яковлева,М., 1981,ч. II.

3. Геометрия /Под ред. Г. Н. Яковлева. М., 1982.

4. Брадис В. М. Четырехзначные математические таблицы (любое издание).

Дополнительная

5. Погорелов А. В, Геометрия 6—10. М., 1983 (и более поздние издания).

6. Валуце И. И., Дилигул Г. Д. Математика для техникумов. М., 1980.

7. Федин Н. Г. Геометрия. М., 1978.

8. Богомолов Н. В. Практические занятия по математике. М., 1983.

9. Пономарев К. К. Курс высшей математики. М., 1974,ч.П.

10. Гмурман В. Е. Теория вероятностей и математическая статистика. М., 1977;

Числовые системы и приближенные вычисления

Действительные числа. Приближенное значение числа. Оценки погрешности приближенного значения чисел Погрешности вычислений с приближенными данными.

Вычисления с помощью микрокалькуляторов и других вычислительных средств.

Развитие понятия числа. Комплексные числа. Действия над комплексными числами. Геометрическая интерпретация комплексных чисел. Тригонометрическая и.показательная формы комплексного числа. Действия над комплексными числами в тригонометрической и показательной формах.

Литература: [2], гл. 1, §  1—4; [7], гл. 1, § 1—5.   .

Производная и ее приложения

Свойства и графики основных элементарных функций. Понятия предела и непрерывности функции в точке. Основные свойства предела.

Предел функции на бесконечности. Вычисление пределов. Производная, ее геометрический и физический смысл. Правила дифференцирования. Дифференциал функции и его геометрический смысл. Приложение дифференциала к приближенным вычислениям. Возрастание и убывание функции. Экстремум функции. Вторая производная и ее физический смысл. Выпуклость, точки перегиба графика функции. Исследование функций и построение графиков. Задачи на наибольшее и наименьшее значение,

Литература: [2], гл. 4, § 15—19; гл. 6, § 27—30; гл. 7, § 31—37; [3], гл. 2, § 4; гл. 5, § 1-17; гл. 6, § 1-15.

Приближенные вычисления

По данной теме сначала изучите § 1—4 гл. 1 [2] или § 1—5 гл, 1 [7]. Затем ознакомьтесь с методическими указаниями по этой теме и внимательно разберите решение примеров из данного пособия. Ответьте на вопросы и выполните упражнения для самопроверки. Решите следующие задачи: [2], гл. 1, № 1.2—1.20 (четные) или [7], гл. 1,№ 1 (6—9), 3,4, 6—16.

Абсолютная и относительная погрешности. При решении практических задач часто приходится иметь дело с приближенными значениями различных величин. К ним относятся: результаты измерений различных величин с помощью приборов; результаты подсчетов большого количества предметов, например числа жителей в городе; значения, полученные при измерениях на графиках, диаграммах, номограммах; проектные данные, нормируемые ГОСТами; результаты округления чисел; результаты действий над приближенными числами; табличные значения некоторых математических величин; результаты вычислений значений функций.

При вычислениях с приближенными числами важно знать отклонение приближенного значения величины от ее точного значения, для чего вводится понятие абсолютной погрешности приближения.

Абсолютной погрешностью Δ (дельта) приближения называется модуль разности между точным значение величины . а и ее приближенным значением х, т. е. | а—х|= Δ.

Например, абсолютная погрешность приближения 0,44 числа 4/9 составляет

.

На практике во многих случаях точное значение величины бывает неизвестно, поэтому абсолютную погрешность приближения найти нельзя. Однако можно дать оценку абсолютной погрешности, если известны приближения с избытком и с недостатком.

Если [ax]<h, или xh <a<x+h, то говорят, что число а равно числу х с точностью до h, и пишут a=x ± h Положительное число h называют границей абсолютной погрешности приближения.

Например, отношение длины окружности к ее диаметру равно иррациональному числу π=3,14159... . Найдем границу абсолютной погрешности общепринятого  приближения числа π числом 3,14:

Δ = |3,14159 ... — 3,141 = 0,00159 ....

За границу абсолютной погрешности приближения можно взять любое число, большее числа 0,00159..., например 0,002, 0,005, 0,01.

Замечание. Граница абсолютной погрешности измерения обычно устанавливается по наименьшему делению прибора (будильник показывает время с точностью до 1 мин, наручные часы с секундной стрелкой показывают время с точностью до 1с).

Цифра α называется: верной, если граница абсолютной погрешности данного приближения не превосходит. единицы того разряда, в котором записана цифра α. В .противном случае цифра называется сомнительной.

Например, в числе а=27,4±0,08 все цифры верные, так как h=0,08<0,1; в числе а= 9,746±-0,04 цифры 9 и 7 верные, поскольку h=0,04<0,1, а цифры 4 и 6 сомнительные, так как h=0,04>0,01.

Замечания. 1. Рекомендуется в записи приближенных чисел сохранять только верные цифры.

2. Если в десятичной дроби последние цифры – нули, то их оставляют в записи числа..

Например, если а =0,26±0,003, то правильная запись числа есть 0,260.

3. Если в целом числе последние нули являются сомнительными цифрами, то их исключают из записи числа.

Например, если а =25000±25, то правильная запись числа есть250 * 102.

4. В записи числа  последняя цифра десятичной записи числа указывает на точность приближения, т. е. граница абсолютной погрешности не превосходит единицы последнего разряда.

Например, запись означает, что а =3,29±0,01.

В десятичной записи числа значащими цифрами числа называют все его верные цифры начиная с первой слева, отличной от нуля.

Например, в числе 1,13 — три значащие цифры, в числе 0,017—две, в числе 0,303 — три, в числе 5,200 —четыре, в числе 25*103—две значащие цифры.

Правило округления чисел. Если первая слева отбрасываемая цифра меньше 5, то округляют с недостатком, а если эта цифра 5 или больше 5, то округляют с избытком.

Например, округляя число 2,783 до сотых, получим 2,78, до десятых — получим 2,8.

Абсолютная погрешность показывает, насколько точным является приближение в случае, когда рассматривается несколько значений одной и той же величины. Лучшим приближением является то, у которого наименьшая абсолютная погрешность.

Однако для сравнения точности приближений разных величин абсолютной погрешности недостаточно. Так, погрешность в 0,5 см слишком велика при измерении толщины книги и допустима при измерении высоты стола, т. е. качество измерения характеризует не сама погрешность, а ее отношение к измеряемой величине.

Относительной погрешностью ω (омега) приближения х величины а называется отношение абсолютной погрешности Δ этого приближения к модулю приближенного значения х, т. е. ω=Δ/|x|. Обычно относительная погрешность выражается в процентах. В расчетах, не требующих высокой точности, бывает достаточно обеспечить относительную погрешность порядка десятых долей процента, что гарантируется при вычислениях с тремя значащими цифрами. Этим объясняется широкое применение в технических расчетах логарифмической линейки.

Так как обычно точное значение величины а, а следовательно, и погрешности Δ неизвестно, то на практике приходится оценивать модуль относительной погрешности некоторым числом ε (эпсилон), которое заведомо не меньше этого модуля: .

В качестве такого числа можно взять отношение h/|x|. Положительное число ε называют границей относительной погрешности.

Пример 1. Сравнить качества измерений толщины книги d (см) и высоты стола Н (см), если известно, d=2±0,5, Н=100±0,5.

Решение. Для сравнения качества измерений найдем относительную погрешность каждого измерения:

, .

Итак, толщина книги измерена с относительной погрешностью до 25%, а высота стола — до 0,5 %. Качество измерения высоты стола намного лучше качества измерения толщины книги.

Действия над приближенными значениями величин. Для того чтобы правильно производить действия над приближенными значениями величин, надо уметь находить погрешности этих действий.

В таблице приведены формулы для оценки границ; погрешностей результатов действий:

Производимое действие

Граница абсолютной погрешности

Граница относительной погрешности

аb

hab=

a+b

a – b

an

Однако строгий учет, погрешности значительно усложняет вычисления, поэтому, если не требуется учитывать погрешности промежуточных результатов, можно пользоваться более простыми правилами.

При сложении и вычитании приближенные вычисления рекомендуется выполнять так:

а) выделить слагаемое с наименьшим числом десятичных знаков;

б) округлить остальные слагаемые так, чтобы каждый из них содержал на один десятичный знак больше, чем выделенное;

в) выполнить действие, учитывая все сохраненные десятичные знаки;

г) результат округлить и сохранить столько десятичных знаков, сколько их в приближенном числе с наименьшим числом десятичных знаков.

Пример 2. Вычислить сумму приближенных чисел 0,6, 0,42 и 0,286. Найти границу погрешности результата.

Решение. Округлим все Данные, сохранив два десятичных знака, и выполним сложение: 0,6+0,42+0,26 ≈ 0,6 + 0,42 + 0,29 = 1,31 ≈ 1,3

Граница погрешности каждого слагаемого не превосходит единицы последнего разряда; тогда h=0,1+0.01+0.001=0.111 ≈ 0.2

Граница погрешности суммы практически мало отличается от границы погрешности наименее точного слагаемого. Итак, сумма равна 1,3±0,2.

При умножении и делении приближенные вычисления рекомендуется выполнять в следующем порядке:

а) выделить исходное данное с наименьшим числом верных значащих цифр;

б) округлить остальные данные так, чтобы каждое из них содержало на одну значащую цифру больше, чем в выделенном;

в) выполнить действия, учитывая все сохраненные значащие цифры;

г) сохранить в результате столько значащих цифр, сколько имеет исходное данное с наименьшим числом верных значащих цифр.

Пример 3. Найти частное приближенных чисел 654,1 и 8,5 и границу погрешности результата.

Решение. Округлим делимое, сохранив три значащие цифры, т. е. до единиц: 654,1 ≈ 654. Выполним деление и вставим в результате две значащие цифры:654,1: 8,5 ≈ 654: 8,5 ≈76,9 ≈77. Чтобы ответить на вопрос, с какой точностью найдено частное, сначала вычислим границу относительной погрешности результата

Границу погрешности частного находим по формуле , т.е. h=77*0,0122=0,9394≈1. Итак, частное равно 77±1.

При возведении в квадрат и в куб в результате сохраняют столько значащих цифр, сколько их имеет основание степени.

При извлечении квадратного и кубического корней в результате сохраняют столько значащих цифр, сколько их имеет подкоренное число.

Пример 4. Вычислить приближенное значение числа 4,132 и найти относительную погрешность вычисления.

Решение. Сначала находим квадрат числа 4,13 и оставляем в результате три значащие цифры 4,132= 17,0569≈17,1. Относительную погрешность вычисления находим по формуле :

, .

Итак, 4,132=17,1 с относительной точностью до 0,5 %.

Замечание. Для более точных вычислений в результатах промежуточных действий рекомендуется сохранять одну запасную цифру, т. е. сохранять на один десятичный знак или на одну значащую цифру больше, чем рекомендует правило.

Пример 5. Вычислить приближенное значение выражения и найти границу погрешности результата.

Решение. Находим значение квадрата числа 5,62 и квадратного корня из числа 18,50; имеем 5,622=31,58; . Теперь получаем

где сначала выполнено деление, а затем умножение.

Найдем границу относительной, погрешности результата

Граница погрешности результата есть h=39,2*0,0058≈0,23≈0,3. Итак, х==39,2±0,3

Пример 6. Вычислить приближенное значение выражения и найти границу погрешности результата.

Решение. Находим значение квадратного корня из числа 6,24 и соз38024'; имеем =2,4980; cos380 24' =0,7837. Далее получим

,

где сначала выполнено деление, а затем умножение.

Найдем границу относительной погрешности результата:

Граница погрешности результата есть . Итак, х=10,04±0,05.

Вопросы и упражнения для самопроверки

1. Назовите источники получения приближенных чисел.

2. Дайте определение абсолютной погрешности приближения.

3. Какое число можно взять за границу абсолютной погрешности?

4. Сформулируйте правило округления чисел.

5. Дайте определение относительной погрешности приближения.

6. Какое число можно взять за границу относительной погрешности?

7. Какие цифры числа называют значащими?

8. Как определяются верные цифры приближенного числа?

9. Сформулируйте правила действий над приближенными числами.

10. Найдите абсолютную погрешность приближенного равенства чисел 11/4 ≈ 0,27

11, Округлите число 73,1729 до тысячных, сотых, десятичных, единиц, десятков, сотен.

12. Округлите число до единиц и найдите абсолютную и относительную погрешности округления:   а)   10,59; б) 0.892.

13. Найдите относительную погрешность приближения: а) числа 1/3 числом 0,33; б) числа 1/7 числом 0,14.

14. Сколько верных цифр имеет число: а) 5,74±0,01 б) 1;174±0,025; в)0,874±0,05?

15. Округлите приближенное значение числа х до первого верного разряда: а)х:=0,2391±0,05; б)х=1,0738±0,0025; в) х=2354±50.

16. Приближенное значение массы Земли равно (5,98±0,01)*1024 кг. Масса пули охотничьего ружья равна (9± I) г. Какое измерение является более точным?

17. Вычислите приближенное значение выражения и границу погрешности результата:

а); б); в).

Ответы. 10. 1/200. 11. 73,173; 83,17; 73,2; 72; 70; 100. 12. а) 11; Δ = 0,41; w = 4%; б) 1; Δ = 0,108; w = 11%. 13. а) 1,1%; б) 2,1%. 14. а) три; б) две; в) одну. 15. а) 0,2±0,09; б) 1,07±0,007; в) 24*102±100. 16. 0,2% < 11%; масса Земли измерена точнее. 17. а) 16±0,6; б) 209±4; в) 1,36±0,02.

Комплексные числа

По данной теме сначала изучите § 1—3 гл. 1 [3] или § 1—9 гл. 9 [7]. Затем ознакомьтесь с методическими указаниями по этой теме и внимательно разберите решение примеров из данного пособия. Ответьте на вопросы и выполните упражнения для самопроверки. Решите следующие задачи: [3], гл. 1, § 3 № 1.3—1.6, 1.25—1:27 (б, в), 1.30 – 1.31 (а, б, в), 1.38—1.40 (а, б) или [7], гл. 9, № 1 – 16.

Из контрольной работы №1 выполните пятое задание своего варианта.

Основные понятия и определения. Решение; задач сводится к решению алгебраических уравнений. Поэтому исследование алгебраических уравнений является одним из важнейших вопросов математики Стремление сделать уравнения разрешимыми — одна из главных причин расширения понятия числа:

Так, для разрешимости уравнения x+a=b положительных чисел недостаточно и приходится вводить отрицательные числа и нуль.

Для решения уравнения недостаточно целых чисел и приходится вводить дробные числа, Целые и дробные числа образуют множество Q иррациональных чисел.

На множестве рациональных чисел разрешимы уравнения вида , однако уравнение х2=2 не имеет рациональных корней. Необходимость решения таких уравнений явилась одной из причин введения иррациональных чисел.

Рациональные и иррациональные числа образуют множество R действительных чисел.

Однако и действительных чисел недостаточно для того, чтобы решить любое алгебраическое уравнение. Например, уравнение х2+1=0 не имеет действительных корней. Поэтому приходится расширять множество действительных чисел до нового множества, такого чтобы в этом множестве уравнения вида х2+ а2 =0 имели решения. Корень уравнения х2+1=0 или х2=-1 называется мнимой единицей и обозначается буквой i. Таким образом, символ i удовлетворяет условию i2= - 1.

Комплексным числом называется выражение вида а+bi, где а и bдействительные числа, а i — мнимая единица.

Число а называется действительной частью комплексного числа, а число bi— мнимой частью.

Комплексное число часто обозначают одной буквой z Множество комплексных чисел принято обозначать буквой С.

Запись комплексного числа в виде z=а+bi называется алгебраической формой записи комплексного числа. Два комплексных числа а1+b1  и а2+b2 называются равными тогда и только тогда, когда а12 и b1= b2, т. е. когда равны их действительные части и коэффициенты при мнимой части.

Понятия «больше» и «меньше» для комплексных чисел не определяются.

Комплексное число z = 0+0i называется нулем и обозначается 0; комплексное число z=a+0i отождествляется с действительным числом a, т. е. а+0i=а; комплексное число z=0 + bi называется чисто мнимым и обозначается  bi, т. е. 0+bi=bi

Число 0 является единственным числом, которое одновременно и действительное, и чисто мнимое.

Комплексные числа а+bi и а — bi называются сопряженными.

Действия над комплексными числами в алгебраической форме. Суммой двух комплексных чисел  и называется комплексное число .

Произведением двух комплексных чисел z1=a1+b1i и z2=a2+b2i называется комплексное число

Вычитание комплексных чисел вводится как операция, обратная сложению; деление комплексных чисел вводится как операция, обратная умножению.

Правила вычитания и деления комплексных чисел, и определяются формулами

;

где a1+b1i0

Формулы, определяющие правила действий над комплексными числами в алгебраической форме, не нуждаются в запоминании.

Формулы суммы, разности и произведения комплексных чисел получаются автоматически, если формально выполнить соответствующие действия над двучленами a1+b1i и a2+b2i и заменить i2 на — 1.

При делении на комплексное число достаточно умножить числитель и знаменатель дроби на число, сопряженное знаменателю, т. е. на a1+b1i 

Возведение комплексного числа в степень производится по формулам возведения двучлена в степень, но при этом надо учитывать, что:

 

Пример 1. Найти сумму и произведение комплексных чисел .

Решение. Сумму находим формальным сложением двучленов 2-7i и 3+5i

Произведение находим формальным перемножением двучленов 2-7i и 3+5i с последующей заменой i2 на -1

Пример 2. Даны комплексные числа и . Найти разность z2-z1 и частное z2/z1.

Решение. Разность находим формальным вычитанием двучленов

Чтобы найти частное z2/z1, умножим числитель и знаменатель этой дроби на число, сопряженное знаменателю z1:

Пример 3. Найти комплексное число

Решение. Выполнив в знаменателе дроби возведение в степень, получим

Умножив числитель и знаменатель полученной. дроби на число, сопряженное знаменателю, получим

Геометрическая интерпретация комплексного числа.

Комплексные числа как и действительные, допускают простую интерпретацию; если вместо координатной прямой использовать координатную плоскость.

Комплексное число изображается на координатной плоскости точкой или вектором , начало которого совпадает с началом координат, а конец с точкой М (рис. 1).

Сама координатная плоскость называется при этом комплексной плоскостью, ось абсцисс — действительной осью, а ось ординат — мнимой осью.

Модулем комплексного числа называется абсолютная величина вектора соответствующего этому числу. Для модуля числа  используются обозначения r, или .

На основании теоремы Пифагора (рис. 1) получается формула

Например, комплексное число имеет модуль, равный 10, так как

Аргументом комплексного числа z0 называется величина угла между положительным направлением действитёльной оси и вектором, соответствующим этому числу (рис. 1).

Для аргумента числа  используются обозначения φ, arg z или arg(a+bi).

Аргумент комплексного числа z0 в отличие от модуля определяется неоднозначно.

Так, аргументами числа 5 является следующие углы:

, , и вообще каждый из углов , ; аргументом числа 3i —следующие углы:

, , (рис. 2) и вообще каждый из углов ,

Любые два аргумента. комплексного числа отличаются друг от друга на слагаемое, кратное

Аргумент комплексного числа можно находить так:

а) найти острый угол ;

б) найти аргумент комплексного числа в зависимости от того, в какой координатной четверти лежит вектор соответствующий этому числу: в I четверти ; во II четверти ; в III четверти ; в IV четверти - .

Пример 4. Найти аргумент комплексного числа

Решение. Находим угол . Вектор, соответствующий данному комплексному числу, лежит в IV координатной четверти (рис.3), поэтому аргументами числа являются каждый из углов .

Аргументы действительных и мнимых чисел надо находить непосредственно, исходя из их геометрической интерпретации, а не используя приведенное выше правило (тем более, что для чисто мнимых чисел это правило вообще нельзя применять).

Тригонометрическая форма комплексного числа.

Пусть дано комплексное число . Из треугольника ΔOMA (см. рис. 1) можно выразить действительные числа а и b через модуль r и аргумент φ числа z следующим образом:

, . Таким образом, комплексное число можно записать в виде

,

где r — модуль комплексного числа, а — один из его аргументов. Представление комплексного числа z0 в указанном виде называется тригонометрической формой записи комплексного числа.

Для того чтобы перейти от алгебраической формы записи комплексного числа к тригонометрической, достаточно найти его модуль и один из аргументов. Аргумент комплексного числа  можно находить из системы.

Пример 5. Записать число в тригонометрической форме. 

Решение. Находим модуль

.

Находим угол

Вектор, соответствующий данному комплексному числу, лежит в III координатной четверти (рис. 4), поэтому одним из аргументов является . Следовательно,

Для того чтобы перейти от тригонометрической формы записи комплексного числа ,к алгебраической, достаточно найти действительные числа а и b по формулам , .

Пример 6. Записать число в алгебраической форме.

Решение. Сначала найдем cos3300 и sin3300

cos 3300=cos (3600-300)=cos300=

sin 3300=sin(3600-300)= - sin300= - 

Тогда , . Следовательно, z=2(cos3300+isin3300)=- i.

Действия над комплексными числами, заданными в тригонометрической форме.

Если и , то

,

Если , то

где — арифметический корень, k=0, 1, 2, …,n – 1.

Пример 7. Даны комплексные числа z1=12(cos2250+isin2250) и z2=3/2(cos750+isin750). Найти их произведение и частное. Ответ записать в алгебраической форме.

Решение. Применяя правила умножения и деления комплексных чисел, имеем ,

Пример 8. Вычислить z=(2(cos240+isin240))5,

Решение. Находим

Пример 9. Вычислить ()10

Решение. Запишем число в тригонометрической форме. Находим

     или φ=3300

Тогда и значит,

=

=

Пример 10. Вычислить. Ответ записать в тригонометрической и алгебраической формах.

Решение. Запишем число -81 в тригонометрической форме. Следовательно

, где k=0,1, 2, 3

При k=0, 1, 2, 3 получим

Показательная форма комплексного числа. Рассматривая функцию у=еx для комплексного переменного, Эйлер установил замечательное соотношение

которое называется формулой Эйлера.

Из этой формулы следует, что каждое комплексное число z можно записать в форме

которая называется показательной формой записи.

Над комплексным числом, заданном в показательной форме, удобно производить умножение и деление, возведение в натуральную степень и извлечение корня:

, k=0, 1, 2, …,(n-1)

Пример 11. Представить число в алгебраической форме

Решение. По условию, r=4, 5/6, откуда

,

Значит, .

Пример 12. Выполнить действия и записать ответ в тригонометрической и показательной формах

Решение. Сначала выполним действия:

Теперь запишем число в тригонометрической и показательной формах, для чего найдем его модуль и аргумент:

Тогда

Вопросы и упражнения для самопроверки

1. Дайте определение комплексного числа.

2. Дайте определение мнимой единицы.

3. Как найти степень мнимой единицы?

4. Какие комплексные числа называются равными; сопряженными?

5. Как изображаются комплексные числа геометрически?

6. Дайте определение модуля и аргумента комплексного числа.

7. Перечислите формы записи комплексного числа.

8. Как выполняются действия над комплексными числами, заданными в алгебраической форме; в тригонометрической форме; в показательной форме?

9. Найдите модуль и аргумент комплексного числа

10. Комплексное число, изображенное точкой (2, —2), запишите во всех трех формах.

11. Запишите в тригонометрической форме комплексные числа: а) ;

б) ; в).

12. Запишите в алгебраической форме комплексные числа:

    а); б); в)

13.Представьте в показательной форме комплексные числа:  

а) ; б)

14. Запишите комплексные числа в алгебраической и тригонометрической формах:

а); б)

Ответы. 9. 4; . 10. 2-2i; ; . 11. а) ;

б) ; в) . 12. а) 3i; б) – 2; в) – 2 – 2i. 13. а) ; б) . 14. а) – 15i; ; б) ; .

Производная и ее приложения

По данной теме сначала изучите§27—30 гл. 6[2] или § 1—7 гл. 5 [7]. Затем ознакомьтесь с методическими указаниями по этой теме и внимательно разберите решение примеров из данного пособия. Ответьте на вопросы и выполните упражнения для самопроверки. Решите следующие задачи: [2], гл. б, § 30, № 6, 21—6.49 (нечетныё) или [7], гл. 5, № 11—31, 36—39, 67, 71 (1—5), 75 (1, 5), 77 (1,2), 79 .(1—4), 81—87.

Из контрольной работы №1 выполните второе и третье задания своего варианта.

Производная. Понятие производной является одним из фундаментальных понятий математики. Многие задачи как самой математики, так и естествознания и техники приводят к этому понятию.

Пусть функция у=f(х) определена в промежутке X. Возьмем из этого промежутка фиксированное значение аргумента х и придадим ему приращение Δх так, чтобы новое значение аргумента х+х принадлежало этому промежутку. Тогда значение функции f(х) заменится новым значением f(х)+у=f(x+x), т. е. функция получит приращение .

Предел отношения приращения функций у к вызвавшему его приращению аргумента х при стремлении х к нулю, т. е.

называется производной функции у=f(х) по аргументу х в точке х.

Производная  обозначается  одним из символов: , ,, , а ее значение при х=х0 обозначается , f(x0), .

Операция нахождения производной называется дифференцированием.

Если функция f(х) имеет производную в точке х, то она называется дифференцируемой в этой точке.

Если функция f(х) имеет производную в каждой точке промежутка X, то говорят, что эта функция дифференцируема на этом промежутке,

Производная сложной функции. Пусть у=f(и), где и является не независимой переменной, а функцией независимой переменной х: и=φ(х). Таким образом,  y=f(φ(х)).

В этом случае функция у называется сложной функцией х, а переменная и –  промежуточным  аргументом.

Производная сложной функции находится на основании следующей теоремы: если y=f(u) и и=φ(х) дифференцируемые функции своих аргументов, то производная сложной функции у=(φ(х)) существует и равна произведению производной функции у по промежуточному аргументу и на производную промежуточного аргумента и по независимой переменной х:

Эта теорема распространяется и на сложные функции, которые задаются с помощью цепочки, содержащёй три звена и более.

Например, если у=f(и), u=φ(v), v=, т. е y=f(φ((х))), то .

Формулы дифференцирования. Во всех приведенных ниже формулах буквами и и v обозначены дифференцируемые функции независимой переменной х: и=и(х), и=v(х), а буквами а, с, п — постоянные:

1. .

2. .

3. .

4. .

5. .

6. .

Остальные формулы записаны как для функций независимой переменной, так и для сложных функций:

7. .   7а. .

8. .  8а. .

9. .  9а. .

10. .  10а. .

11. .  11а.

12. .  12а. .

13. .  13а. .

14. .  14а. .

15. .  15a. .

16.   16a. .

17.   17a. , где а > 0, а0.

При решении приведенных ниже примеров сделаны подробные записи. Однако следует научиться дифференцировать без промежуточных записей.

Пример 1. Найти производную функции

Решение. Данная функция есть алгебраическая сумма функций. Дифференцируем ее, используя формулы 3, 5, 7 и 8:

Пример 2. Найти производную функции .

Решение. Применяя формулы 6, 3, 7 и 1, получим

.

Пример 3. Найти производную функции и вычислить ее значение при

Решение. Это сложная функция c промежуточным аргументом sin. Используя формулы 7а и 10, имеем

.

Вычислим значение производной при

.

Пример 4. Найти производную функции .

Решение. Это сложная функция с промежуточным аргументом соs х, Применяя формулы 3, 5, 7а, 11, 16а, получим

.

Пример 5. Найти производную функции

Решение. Дифференцируем данную функцию по формулам 6, 12, 3 и 1:

.

Пример 6. Найти производную функции и вычислить ее значение при t=2.

Решение. Сначала преобразуем функцию, используя свойства логарифмов:

Теперь дифференцируем по формулам 3, 16а, 7 и 1:

.

Вычислим значение производной при t=2

.

Пример7. Найти производную функции и вычислить ее значение при х=0. Решение. Используем формулы 6, 3, 14а, 9а, 5 и 1:

Вычислим значение производной при х=0:

 

Геометрический смысл производной. Производная функции имеет простую и важную геометрическую интерпретацию.

Если функция у=f(х) дифференцируема в точке х, то график этоq функции имеет в соответствующей точке касательную, причем угловой коэффициент касательной равен значению производной в рассматриваемой точке.

Угловой коэффициент касательной, проведенной к графику функции у=f(х) в точке 0, у0), равен значению производной функции при х = х0, т.е. .

Уравнение этой касательной имеет вид

.

Пример 8. Составить уравнение касательной к графику функции в точке A(3, 6).

Решение. Для нахождения углового коэффициента касательной найдем производную данной функции:

.

Угловой коэффициент касательной равен значению производной функции при x=3:

.

Уравнение касательной имеет вид

, т.е. 10x-y-24=0

Пример 9. Составить уравнение касательной, проведенной к графику функции в точке с абсциссой х =2.

Решение. Сначала найдем ординату точки касания А(2, у). Так как точка А лежит на кривой, то ее координаты удовлетворяют уравнению кривой, т.е.

; А(2; 2).

Уравнение касательной, проведенной к кривой в точке А(2, 2), имеет вид y-2=k(x-2). Для нахождения углового коэффициента касательной найдем производную:

Угловой коэффициент касательной равен значению производной функции при х=2:

.

Уравнение касательной таково: y-2=-(x-2), y-2=-x+2,т.е. x+y-4=0.

Физический смысл производной. Если тело движется по прямой по закону s=s(t), то за промежуток времени (от момента t до момента t+) оно пройдет некоторый путь. Тогда есть средняя скорость движения за промежуток времени .

Скоростью движения тела в данный момент времени t называется предел отношения приращения пути к приращению времени , когда приращение временя стремится к нулю:

.

Следовательно, производная пути s по времени t равна скорости прямолинейного движения тела в данный момент времени:

.

Скорость протекания физических, химических и других процессов также выражается с помощью производной.

Производная функции y=f(x) равна скорости изменения этой функции при данном значении аргумента х:

Пример 10. Закон движения точки по прямой задан формулой s=5t3-3t2+4 (s — в метрах, t — в секундах). Найти скорость движения точки в конце первой секунды.

Решение. Скорость движения точки в данный момент времени равна производной пути s по времени .

Итак, скорость движения точки в конце первой секунды равна 9 м/с.

Пример 11. Тело, брошенное вертикально вверх, движется по закону, где — начальная скорость, g — ускорение свободного падения тела. Найти скорость этого движения для любого момента времени t. Сколько времени будет подниматься тело и на какую высоту оно поднимется, если = 40 м/с?

Решение. Скорость движения точки в данный момент времени t равна производной пути s по времени t:

.

В высшей точке подъема скорость тела равна нулю:

, , ,  t=4.1 c

За 40/g секунд тело поднимется на высоту

, м.

Вторая производная. Производная функции у=f(х) в общем случае является функцией от х. Если от этой функции вычислить производную, то получим производную второго порядка или вторую производную функции y=f(x).

Второй производной функции у=f(x) называется производная от ее первой производной у'=f'(х).

Вторая производная функции обозначается одним из символов — у", f"(х),. Таким образом, .

Аналогично определяются и обозначаются производные любого порядка. Например, производная третьего порядка:

или , .

Пример 12. Найти вторую производную функции

Решение. Сначала найдем первую производную .

Дифференцируя еще раз, найдем вторую производную:

.

Пример 13. Найти вторую производную функции и вычислить ее значение при

х = 2.

Решение. Сначала найдем первую производную:

Дифференцируя еще раз, найдем вторую производную:

.

Вычислим значение второй производной при х = 2; имеем y”(2)=4/(2-1)3=4/1=4.

Физический смысл второй производной. Если тело движется прямолинейно по закону s=s(t), то вторая производная пути s по времени t равна ускорению движения тела в данный момент времени t:

Таким образом, первая производная характеризует скорость некоторого процесса, а вторая производная— ускорение того же процесса.

Пример 14. Точка движется по прямой по закону s= t - sint. Найти скорость и ускорение движения при t=/2.

Решение. Скорость движения тела в данный момент времени равна производной пути s по времени t, а ускорение — второй производной пути s по времени t. Находим:

; тогда .

; тогда .

Пример 15. Скорость прямолинейного движения пропорциональна квадратному корню из пройденного пути (как, например, при свободном падении). Доказать, что это движение происходит под действием постоянной силы.

Решение. По закону Ньютона, сила F, вызывающая движение, пропорциональна ускорению, т. е. F=ka(t)  или F=ks”(t).

Согласно условию, v(t)=s`(t)=. Дифференцируя это равенство, найдем

.

Следовательно, действующая сила .

Приложения производной к исследованию функций

Условие постоянства функции. Дифференцируемая функция у=f(х) постоянна на промежутке X тогда и только тогда, когда внутри X.

Условие возрастания функции. Дифференцируемая функция у=f(х) монотонно возрастает на промежутке X тогда и только тогда, когда ее производная не отрицательна внутри этого промежутка: , причем производная f'(х) обращается в нуль в конечном числе точек, лежащих внутри промежутка X.

Это условие геометрически означает, что касательная к графику монотонно возрастающей функции образуется положительным направлением оси Ох острый угол или параллельна ей (рис. 41).

Условие убывания функции. Дифференцируемая функция y=f(x) монотонно убывает на промежутке X тогда и только тогда, когда ее производная не положительна внутри этого промежутка: f’(x)0, причем производная f'(х) обращается в нуль в конечном числе точек, лежащих внутри X.

Это условие геометрически означает, что касательная к графику монотонно убывающей функции образует с положительным направлением оси Ох тупой угол или параллельна ей (рис. 42).

Экстремумы функции. Говорят, что функция у=f(х) имеет максимум в точке х1 (рис.43), если значение функции в этой точке больше, чем ее значения во всех точках, достаточно близких к x1, т.е. если f(x1+)<f(x1) для любых , как положительных, так и отрицательных, но достаточно малых по модулю.

Таким образом, х=х1 — точка максимума, а ymax=f(x1) – максимум функции.

Говорят, что функция у=f(х) имеет минимум в точке х2 (рис.43), если значение функции в этой точке меньше, чем ее значения во всех точках, достаточно близких к x2, т.е. если f(x2+)>f(x2) для любых , как положительных, так и отрицательных, но достаточно малых по модулю. Таким образом, х=х2 — точка минимума, а ymin=f(x2) – минимум функции.

Если в некоторой точке, функция имеет максимум или минимум, то говорят, что в этой точке имеет место экстремум. Значение функции в этой точке называется экстремальным.

Замечание. Следует помнить: 1) что максимум (минимум) не является обязательно наибольшим (наименьшим) значением, принимаемым функцией; 2) функция может иметь несколько максимумов или минимумов; 3) функция, определенная на отрезке, может достигать экстремума только во внутренних точках этого отрезка.

Необходимое условие экстремума. Если функция у=f(х) имеет экстремум при х=х0, то ее производная в этой точке равна нулю или бесконечности либо вовсе не существует при этом сама функция в точке х0 определена.

Из этого следует, что точки экстремума функции следует разыскивать только среди тех, в которых ее первая производная равна нулю, или бесконечности, или не существует. Эти точки называются критическими точками I рода.

Этот признак экстремума является только необходимым. Поэтому, определив критические точки I рода, надо каждую из них в отдельности исследовать на основании достаточных условий экстремума.

Первое достаточное условие существования экстремума функции. Пусть точка х=х0 является критической точкой I рода функции у=f(х), а сама функция дифференцирует во всех точках некоторого промежутка, содержащего эту точку (за исключением, возможно, самой этой точки). Тогда:

1) если при переходе слева направо через критическую точку I рода х=х0 первая производная меняет знак с плюса на минус, то в этой точке функция достигает максимума, т.е. х=х0точка максимума, ymax=f(x0);

2) если при переходе слева направо через критическую точку I рода х=х0 первая производная меняет знак с минуса на плюс, то в этой точке функция достигает минимума, т.е. х=х0 точка минимума, ymin=f(x0);

3} если при переходе через критическую точку I рода первая производная не меняет знака, то в этой точке экстремума нет.

Для исследования функции на экстремум по первой производной следует:

1. Найти область определения функции.

2. Найти первую производную функции и критические точки I рода.

3. Отметить границы области определения и критические точки I рода на числовой прямой.

4. Исследовать знак производной в каждом из полученных интервалов,

5. Выписать точки экстремума и вычислить экстремумы функции.

Пример 16. Найти экстремумы функции у=(1-х2)3.

Решение. 1. Областью определения функции служит множество всех действительных чисел, т.е. xR

2. Функция имеет производную всюду, поэтому определяем критические точки из условия f`(x)=0. Находим производную:

;

; .

3. Отмечаем эти критические точки на числовой прямой (рис. 44).

4. Исследуем знак производной у=-6x(1-x2)2 в каждом из полученных интервалов: у' (— 2)>0, у'(-0,5)>0, y'(0,5)<0, y'(2)<0.

5. Точка х = 0 — точка максимума, так как при переходе через нее слева направо производная меняет знак с плюса на минус: ymax=y(0) = 1. Точки х= — 1 и x=1 не являются точками экстремума.

Второе достаточное условие существования экстремума функции. Если в точке х=х0 первая производная функции равна нулю (f'(х0)=0), а вторая производная отлична от нуля, то х =х0 точка экстремума.

При этом если вторая производная в этой точке положительна (f”(x0)>0), то х=х0 — точка минимума; если вторая производная в этой точке отрицательна (f”(х0)<0), то х=х0 точка максимума.

Для исследования функции на экстремум по первой и второй производной следует:

1. Найти область определения функции.

2. Найти первую производную функции и стационарные точки, т.е. точки, в которых она обращается в нуль.

3. Найти вторую производную функции и исследовать ее знак в каждой стационарной точке.

4. Выписать точки экстремума и вычислить (если нужно) экстремумы функции.

Пример 17. Найти экстремумы функции f(х)=х3 Зх2+1.

Решение. 1. Областью определения функции служит множество всех действительных чисел, т.е. xR

2. Функция имеет производную всюду, поэтому критические точки определяем из условия f`(x)=0:

, , , , .

3. Находим вторую производную функции f"(х) =6x—6. Исследуем знак второй производной в каждой критической точке: f"(0) = - 6<0; значит, х=0 — точка максимума, ymax=y(0) = 1.

f"(2)=6>0; значит, х=2 — точка минимума, ymin=y(2)=23-3*22+1=8-12+1= - 3.

Наибольшее и наименьшее значения функции. Наибольшим значением функции называется самое большое, а наименьшим значением — самое меньшее из всех ее значений.

Функция может иметь только одно наибольшее значение и только одно наименьшее значение или может не иметь их совсем,

Нахождение наибольшего и наименьшего значений непрерывных функций основывается на следующих свойствах этих функций:

1) если в некотором открытом промежутке а<х<.Ь (конечном или бесконечном) функция у=f(х) непрерывна и имеет только один экстремум и если это максимум, то он и является наибольшим значением функции, а если минимум — наименьшим значением функции в этом промежутке;

2) если функция у=f(х) непрерывна на отрезке , то она обязательно имеет на этом отрезке наибольшее и наименьшее значения. Эти значения достигаются ею или в точках экстремума, лежащих внутри отрезка, или на концах этого отрезка.

Поэтому, чтобы найти наибольшее и наименьшее значения отрезке где она непрерывна, следует:

1. Найти экстремумы функции на данном отрезке.

2. Найти значения функции на концах отрезка: f(а) и f(b)

3. Из всех найденных значений выбрать наибольшее и наименьшее,

Пример 18. Найти наибольшее и наименьшее значения функции на отрезке 2x4

Решение. 1. Найдем экстремумы функции, для чего найдем производную функции и критические точки I рода из условия :

;

при , , , .

Отметим критические точки 1 рода х=—1, x=0, x = 3 на числовой прямой (рис. 45),

Исследуем знак производной в каждом из полученных интервалов: y`=(—2)<0, у'(—0,5) >0, y`(1)<0, у'(4)>0. Таким образом,

; ;

2. Найдем значения функции на концах отрезка;

;

3. Итак, наибольшее значение функции yнаиб=у(-2)=5 , а наименьшее значение функции унаим=у(3)=-9.

Пример 19. Требуется изготовить закрытый цилиндрический бак объемом 250π см3. Какими должны быть его размеры, чтобы на его изготовление пошло наименьшее количество материала?

Решение. Здесь требуется определить радиус основания R и высоту Н цилиндра, чтобы при заданном объеме площадь его полной поверхности была наименьшей.

Площадь полной поверхности цилиндра вычисляется по формуле s=2πRH+2πR2.

Наименьшее значение этой функции и следует определить. Так как s является функцией двух независимых переменных, то одну из них надо исключить. Известно, что объем цилиндра VR2H или V = 250π. Выразим Н через V:

. Тогда ,

1. Областью определения функции s являются положительные значения радиуса, т.е.

2. Находим производную: ; при , , R = 5.

3, Находим вторую производную;

.

Так как S"(5)>0, то при R = 5 имеет место минимум функции S, который и является наименьшим значением функции S. Тогда H=250/R2, или H = 250/25=10.

Итак, на изготовление цилиндрического бака пойдет наименьшее количество материала, если длина радиуса основания цилиндра равна 5 см, а высота цилиндра 10 см.

Пример 20. Требуется изготовить ящик с крышкой, стороны основания которого относятся как 1 :2, а площадь полной поверхности 108 см2. Какими должны быть его размеры, чтобы его объем был наибольшим?

Решение. Здесь требуется определить стороны основания а и b и высоту H прямоугольного параллелепипеда, чтобы при заданной площади полной поверхности его объем был наибольшим.

По условию, a:b=1:2, откуда а=х, b=2х. Объем прямоугольного параллелепипеда равен V = аbН или V=2Н. Надо исключить переменную Н. Известно, что S = 108 и S = 2SОСн+Sбок= Росп-Н, Имеем

, , ,

, . Тогда .

Наибольшее значение этой функции и следует определить.

1. Областью определения функции V являются положительные значения х, т. е. x:>0.

2. Находим производную: ; при , , .

3. Находим вторую производную: = - 8x;V”(3) <0, т. е. х = 3 функция имеет максимум, который и служит наибольшим значением функции. При этом .

Итак, объем ящика является наибольшим, если сторона его основания имеют длину 3 и 6 см, а высота 4 см.

Пример 21. Число 10 разбить на два положительных слагаемых так, чтобы сумма их кубов была наименьшей.

Решение. Пусть одно из слагаемых равно х, тогда другое слагаемое есть 10 — х. Сумма кубов этих слагаемых равна ,

Наименьшее значение этой функции и надо определить.

1. Областью определения функции 5 являются положительные значения я, т. е. х>0,

2. Находим производную:

; при , , х = 5.

3. Находим вторую производную: s"=60, s"(5)>0, т. е. при х=5 функция 5 имеет минимум, который и является наименьшим значением функции.

Итак, число 10 надо разложить на два равных слагаемых: 5 и 5.

Пример 22. Закон прямолинейного движения тела задан уравнением s=-t3+9t2-24t-8 (s - в метрах, tв секундах). Найти максимальную скорость движения тела.

Решение. Скорость движения тела в данный момент времени равна производной пути s по времени t:

Исследуем эту функцию на экстремум с помощью второй производной: v`(t)=-6t+18, v”(t)=-6. Вторая производная отрицательна; следовательно, скорость является наибольшей при t=3.

Максимальная скорость движения составляет , м/с.

Направление вогнутости точки перегиба кривой. Говорят, что на промежутке а<х<b кривая обращена выпуклостью вверх или выпукла () если она лежит ниже касательной, проведенной в её точке (рис. 46).

Говорят, что на промежутке b<х<с кривая обращена выпуклостью вниз или вогнута (), если она лежит выше касательной, проведенной в любой ее точке (рис.46).

Точкой перегиба непрерывной кривой называется точка А .(рис. 46), при переходе через которую кривая меняет свою вогнутость на выпуклость или наоборот.

Достаточное условие выпуклости (вогнутости) кривой. График дифференцируемой функции у=f(х) является выпуклым на промежутке а<х<b, если вторая производная функция отрицательна в каждой точке этого промежутка: f”(х)<0 при a<x<b

График дифференцируемой функции у=f(х) является вогнутым на промежутке b<х<с, если вторая производная функции положительна в каждой точке этого промежутка: f”(х)>0 при b<.х<с.

Точки, в которых вторая производная функции равна нулю, или бесконечности, или не существует, называются критическими точками II рода.

Если при переходе через критическую точку II рода х=х0 вторая производная функции меняет знак, то х=х0 — абсцисса точки перегиба. Ордината точки перегиба равна значению функции в точке х0. Точка .{х0, f(х0)) — точка перегиба графика функции у=f(х).

Чтобы найти направление вогнутости и точки перегиба кривой, следует:

1. Найти область определения функции.

2. Найти вторую производную функции и критические точки II рода.

3. Отметить границы области определения и критические точки II рода на числовой прямой.

4. Исследовать знак второй производной в каждом из полученных интервалов.

5. Записать промежутки выпуклости и вогнутости, абсциссу точки перегиба и вычислить ее ординату.

Пример 23. Определить направление вогнутости и точки перегиба кривой y=x4+2x3-12x2-5x+2.

Решение. 1. Областью определения функции служит множество всех действительных чисел, т.е. xR

2. Найдем вторую производную функции и критические точки II рода из условия y"=0:

; ;

; при , , , .

3. Отметим критические точки II рода х=-2 и х = 1 на числовой прямой (рис. 47).

4. Исследуем знак второй производной в каждом из полученных интервалов;

у"(— 3)>0, у”(0)<0, у"(2)>0.

5. Кривая вогнута при x< - 2 и x;>1; кривая выпукла при — 2<x<1;

xтп= - 2, утп=у(-2)=16-2*8-12*4+5*2+2=-36;  

хтп=1, утп=у(1)=1+2-12-5+2=-12

Точки перегиба (— 2, —36), (1, —12).

Общая схема исслёдования функции построения их графиков.

1. Найти область определения функции и поведение функции в границах области определения,

2. Выяснить вопрос о четности, нечетности и периодичности функции.

3. Найти точки пересечения графика функции с осями координат.

4. Найти промежутки монотонности и экстремумы функции.

5. Найти направление вогнутости и точки перегиба графика функции.

6. Построить график функции, используя все полученные результаты исследования. Если их окажется не-достаточно, то следует найти еще несколько точек графика функции, исходя из ее уравнения,

Построение графика функции целесообразно выполнять по его элементам, вслед за выполнением отдельных пунктов исследования.

Пример 24. Построить график функции у

Решение. 1. Областью определения функции служит множество всех действительных чисел, т.е. xR

Далее, находим ,

2. Выясняем вопрос о четности или нечетности функции:

, , .

Значит, функция не является не четной, ни нечетной.

3. Найдем точки пересечения графика функции с осями координат:

  .

4- Находим промежутки монотонности и экстремумы функции:

, при и .

Отметим критические точки I рода x=0 и x:=3 на числовой прямой (рис. 48) и исследуем знак производной в каждом из полученных интервалов: у'(—1)>-0,

Функция возрастает при x<3 и убывает при x>3; x=3— точка максимума,

.

5. Находим направление вогнутости и точки перегиба графика функции:

.

Итак, y" = 0 при x1= 0, x2=2.

Отметим критические точки II рода x=0 и x=2 на числовой прямой (рис. 49) и исследуем знак второй производной в каждом из полученных интервалов: у"(—1)<0, y”(1)>0, y”(3)<0

График функции является выпуклым при x<:0 x>2 и вогнутым при 0<х<2:

.

Точки перегиба графика функции (0, 0) и (2;3,2). Отметим все полученные точки в системе координат и соединим их плавной кривой (рис. 50).

Для уточнения графика найдем дополнительную точку у=(-1)=-1

Вопросы и упражнения  для  самопроверки

1. Дайте определение производной функции,

2. В чем состоит геометрический смысл производной?

3. В чем состоит физический смысл производной?

4. Дайте определение второй производной  функции.

5. В чем состоит физический смысл второй производной?

6. Напишите все формулы дифференцирования.

7. Сформулируйте условие постоянства функции,

8. Сформулируйте условия возрастания и убывания функции.

9. Сформулируйте необходимое условие существования экстремума функции.

10. Сформулируйте достаточные условия существования экстремума функции.

11. Как найти точки экстремума и экстремум функции?

12. Как найти наименьшее и наибольшее значения функций на отрезке?

13. Сформулируйте условия выпуклости и вогнутости кривой!

14. Как найти направление вогнутости и точки перегиба кривой?

15. Найдите производные функций: а) у=(2 – z2)4; б) y = ln sin3 (1—х); в) f(х)=cosх.

16. Найдите вторую производную функций: а) ; б); в)y=cos2x

17. Составьте уравнение касательной к кривой y=tg2x в начале координат.

18. При каком значении переменной х касательные к кривым у=х2 и у=х3 параллельны?

19. Тело движется прямолинейно по закону s= (s—в метрах, t — в секундах). Найдите скорость движения в тот момент времени, когда ускорение равно нулю.

20. Найдите экстремумы функции y=2x3-6x2-18x+7

21. Найдите наибольшее и наименьшее значения функции у=х3Зх29х+35 на отрезке — 4x4.

22. Кусок проволоки длиной в 84 см требуется согнуть в виде прямоугольника так, чтобы площадь этого прямоугольника была наибольшей.

23. Число 16 разложите на два таких положительных множителя, чтобы сумма их квадратов была наименьшей.

24. Определите направление вогнутости и точки перегиба кривой у = Зх55x4+4;

Дифференциал функции

По данной теме сначала изучите §4 (1 – 3) гл. 2 [3] или § 1 – 5 гл. 6 [7]. Затем ознакомьтесь с методическими указаниями по этой теме и внимательно разберите решение примеров из данного пособия. Ответьте на вопросы и выполните упражнения для самопроверки. Решите следующие задачи: [3], гл. 2, §4, № 2.1 – 2.2 или [7], гл. 6, № 1 – 16.

Из контрольной работы №1 выполните четвертое задание своего варианта.

Понятие дифференциала функции. С понятием производной тесно связано важное понятие математики — понятие дифференциала.

Пусть у=f(х) есть некоторая функция, имеющая в определенной точке х производную f'(х). Дадим аргументу х приращение , тогда функция получит приращение

По определению производной имеем . Так как разность между переменной, имеющей предел, и этим пределом является бесконечно малой, то есть величина бесконечно малая при :

, где при

Тогда у=f'(x)+a()

Как видно, если функция у=f(х) имеет производную в точке х, то приращение функции в этой точке состоит из двух слагаемых. Второе слагаемое α(х  как произведение двух бесконечно малых величин есть бесконечно малая более высокого порядка, чем . Если f'(х)= 0, то первое слагаемое f'(x) имеет тот же порядок, что и . значит, при малых второе слагаемое менее важно, чем первое. Это первое слагаемое (независимо от того, будет ли f`(х)=0) и называют дифференциалом.

Дифференциалом функции у=f(х) в точке х называется главная часть f'(х)  приращения функции, линейно зависящая от приращения аргумента .

Дифференциал обозначается символом dу. По определению, dу=f'(х)dх.

В частности, при f(х)=х получим dx=1* или dx=, т. е. дифференциал аргумента равен его приращению.

Тогда dy=f'(х)dх, т. е. дифференциал функции у=f(x) в точке х равен произведению производной в точке х на дифференциал аргумента.

Отсюда  , так что выражение, которое мы раньше считали цельным символом, теперь можно рассматривать как дробь, равную отношению дифференциала функции к дифференциалу аргумента.

Нахождение дифференциала функции называется дифференцированием, так же как и нахождение производной.

Геометрически приращение функции равно приращению KN ординаты точки кривой, а дифференциал функции dу равен соответственному приращению КР ординаты касательной, проведенной к кривой в точке (х,f(x)), когда аргумент получает приращение Δх (рис.51).

Пример 1. Найти дифференциал функции y=(2x3-4)5

Решение. Находим производную данной функции:

.

Умножив производную на дифференциал аргумента, получим дифференциал функции: dy=30x2(2x3-4)4dx.

Пример 2. Найти дифференциал функции s

Решение. Сначала найдем производную данной функции

.

Умножив производную на дифференциал аргумента, получим дифференциал функции:

.

Пример 3. Вычислить значение дифференциала функции при φ=π/18, 0,01.

Решение. Дифференциал функции вычислим по формуле dv=v`()d. Прежде чем применить эту формулу, найдем производную функции и ее значение при

;

; .

Следовательно, dv = —3,375 • 0,01 = —0,03375.

Приложение дифференциала к приближенным вычислениям. Приращение функции связано с дифференциалом функции соотношением , где при .

Приращение функции и дифференциал функции отличаются друг от друга на бесконечно малую, имеющую порядок малости выше чем . Если пренебречь этой бесконечно малой, то получим приближенное равенство , т.е. при малых приращениях аргумента приращение функции с достаточной степенью точности можно заменить ее дифференциалом.

Учитывая, что , получим , откуда .

Эти приближенные равенства применяются для приближенных вычислений, так как вычисление дифференциала функции значительно проще, чем вычисление ее приращения.

Пример 4. Вычислить приближенное значение приращения функции y=x2+2x+5 при изменении аргумента от х=2 до x=2,001.

Решение. Находим дифференциал аргумента: .

Приращение аргумента мало, поэтому приращение функции приближенно равно ее дифференциалу dy. Дифференциал функции вычисляем по формуле dу=у'(х)dх. Предварительно найдем производную функции и ее значение при х=2:

, . Тогда и .

Точное значение приращения функции найдем по формуле , где

,

Тогда = 13,006001 —13 = 0,006001.

Сравнивая полученный результат с дифференциалом dy, видим, что абсолютная погрешность равна 0,000001. Однако абсолютная погрешность не дает достаточно полной характеристики точности подсчета. Поэтому вычислим и относительную погрешность:

.

Эта точность обычно оказывается вполне достаточной для расчетов, производимых в технике. Поэтому такой погрешностью можно пренебречь и вместо приращения функции находить ее дифференциал, который вычислить проще, так как он зависит от линейно.

Пример 5. Вычислить приближенное значение функции у = x3+x2 – 2x при х= 2,01.

Решение. Найдем дифференциал аргумента: dх==2,01 — 2=0,01. Приращение аргумента мало, поэтому для вычисления приближенного значения функции воспользуемся формулой

или .

Сначала найдем значение функции при х=2: f(2) = 23+22— 4 = 8.

Дифференциал находим по формуле dy=y`(x)dx, для чего найдем производную функции и ее значение при x=2:

, .

Тогда dy=1,4*0.01=0,14

Следовательно, f(2,01)8+0,14=8,14

Пример 6. Найти приближенное значение .

Решение. Нам надо найти приближенное значение функции при x = 16,06.

Найдем дифференциал аргумента: dх==16,06—16 = 0,06. Приращение аргумента мало, поэтому f(16,06).

Дифференциал находим по формуле dy=y`(x)dx, для чего сначала найдем производную функции и ее значение при х = 16.

; .

Тогда dy=0,125-0,06 = 0,0075.

Следовательно, . В действительности

Пример 7. Найти приближенное  значение .

Решение. Как и в предыдущем примере, имеем:

;

, ;

; , ; .

Тогда .

Пример 8. Объем куба, ребро которого равно 4 см, при нагревании увеличился на 0,96 см3. Как при этом увеличилось ребро куба?

Решение. Объем куба с ребром х вычисляется по формуле V = x3. Так как , то dV0.96.

Дифференциал функции вычисляем по формуле dV=V`(x)dx, откуда .

Прежде чем воспользоваться этой формулой, найдем производную функции и ее значение при х = 4: V`=3x2, V`(4)=3*42=48.

Теперь находим , , т.е. ребро куба увеличилось приблизительно на 0,02 см.

Вопросы и упражнения для самопроверки

1. Дайте определение дифференциала функции.

2. Чему равен дифференциал независимой переменной (аргумента)?

3. По какому правилу находят дифференциал функции?

4. В чем состоит геометрический смысл дифференциала функции?

5. Как применяют дифференциал функции в приближенных вычислениях?

6. Найдите дифференциалы функций: а) ; б) .

7. Вычислите значение дифференциала функции s= при изменении t от 4 до 4,025.

8. Вычислите приближенное значение приращения функции у = (1+х—х2)3 при изменении аргумента от 3 до 2,998.

9. Найдите приближенное значение: а); б).

10. Объем куба, ребро которого равно 40 см, при нагревании увеличился на 0,05 своего первоначального значения. Найдите удлинение ребра куба.

11. Сторона основания правильной четырехугольной призмы равна 12 см, а высота 30 см. Насколько (приближенно) уменьшится ее объем, если, не меняя высоты, сторону основания уменьшить на 0,01 см?

Ответы. 6. а) ; б) . 7. 0,02. 8. 0,75. 9. а) 1,0025; б) 0,994. 10. см. 11. 7,2 см2.

КОНТРОЛЬНАЯ РАБОТА№1

Интеграл и его приложения

Первообразная. Неопределенный интеграл и его свойства. Основные табличные интегралы. Интегрирование подстановкой.

Определенный интеграл и его геометрический смысл. Основные свойства и вычисление определенного интеграла.

Вычисление площадей фигур с помощью определенного интеграла. Приближенное вычисление определенного интеграла (методы прямоугольников и трапеций), Применение интеграла к решению физических задач.

Литература: [3], гл. 2, § 5 (1—3), § 6 (1, 2); гл. 3, § 7—10 (1, 2), 11 (1, 2); гл. 4, § 12, 14 (1—3); [7], гл. 7, § 1—6; гл. 8, § 1—14.

Из контрольной работы №2 выполните первое, второе и третье задание своего варианта.

Дифференциальные уравнения

Задачи, приводящие к дифференциальным уравнениям. Понятие о дифференциальном уравнении первого порядка. Задача Коши. Дифференциальные уравнения с разделяющимися переменными. Линейные дифференциальные уравнения первого порядка.

Дифференциальные уравнения второго порядка. Задача Коши. Линейные однородные дифференциальные уравнения второго порядка с постоянными коэффициентами. Дифференциальные уравнения в науке и технике.

Литература: [3], гл. 7, § 24—30, [7], гл. 10, § 1, 2.

Из контрольной работы №2выполните четвертое задание своего варианта.

Элементы теории вероятностей

Предмет теории вероятностей. Понятие о случайном опыте и случайном событии. Относительная частота события. Определение вероятности события.

Основные понятия комбинаторики.

Операции над событиями. Теорема сложения вероятностей. Независимые события. Условные вероятности. Теорема умножения вероятностей, Формула полной вероятности.

Дискретная случайная величина, закон ее распределения. Биномиальное распределение, формула Бернулли.

Математическое ожидание и дисперсия дискретной случайной величины. Понятие о законе больших чисел. Понятие о задачах математической статистики.

Литература: [3], гл. 6, § 18 (1—3); гл. 7, § 20— 23; [7], гл. 11,§ 1-9.

Из контрольной работы №2 выполните шестое задание своего варианта.

Методические указания

Неопределенный интеграл

По данной теме сначала изучите § 5 (1—3), 6(1, 2) гл. 2 [3] или § 1 — 6 гл. 7 [7]. Затем ознакомьтесь с методическими указаниями по этой теме и внимательно разберите решение примеров из данного пособия, Ответьте на вопросы и выполните упражнения для самопроверки. Решите следующие задачи: [3], гл. 2, № 2.4 (1—14, 26, 33—54) или[7], гл. № 1 — 19,33—36,37—51.

Из контрольной работы №2выполните первое задание своего варианта.

Понятие неопределенного интеграла. Напомним, что дифференцирование — это действие, с помощью которого по данной функции находится ее производная или дифференциал. Например, если F(х)=х5, то F' (х) =5х4.

Как мы знаем, нахождение производной имеет большое практическое значение. Так, по данному закону движения тела s = s(t) мы путем дифференцирования находили скорость v(t)=st, а затем и ускорение а(t)=st по данному уравнению кривой у=F(х) определяли угловой коэффициент касательной, проведенной к этой кривой: k = f'(х).

На деле, однако, часто приходится решать обратную задачу: по известной скорости движения тела устанавливать закон его движения, по данному угловому коэффициенту касательной к кривой находить уравнение этой кривой и т. п.( иначе говоря, по данной производной отыскивать функцию, от которой найдена эта производная, т. е. выполнять действие, обратное дифференцированию. Это действие называется интегрированием. С помощью интегрирования по данной производной или дифференциалу функции находится сама функция. Например, если F'(х)=9x8, то F(х)=х9, так как (х9)'=9x8.

Дифференцируемая функция F(х), а<х<b называется первообразной для функции f(х) на интервале а <х<b, если F'(х)=f(х) для каждого а<х<Ь.

Так, для функции f(x)=cos x первообразной служит функция F(х)=sin x, поскольку (sinх}'=соs х.

Для заданной функции ее первообразная определяется неоднозначно.

Справедлива теорема: если F(х)—первообразная для f(х) на некотором промежутке, то и функция F(х)+С, где С — любая постоянная, также является первообразной для функции f(х) на этом промежутке. Обратно; каждая функция, являющаяся первообразной для f(х) в данном промежутке, может быть записана в виде F(x)+C

Значит, достаточно найти для данной функции f(х) только одну первообразную функцию F(х), чтобы знать все первообразные, так как они отличаются друг от друга только на постоянную величину.

Совокупность F(х)+С всех первообразных функций f(х) на интервале а<х<b называют неопределенным интегралом от функции f(х) на этом интервале и пишут . Здесь f(х}dх—подынтегральное выражение; f(х) — подынтегральная функция; х — переменная интегрирования; С—произвольная постоянная.

Например , так как (tgx+c)`=

Если функция f(х) имеет на некотором промежутке хотя бы одну первообразную, то ее называют интегрируемой на этом промежутке. Можно доказать, что любая функция, непрерывная на отрезке аb, интегрируема на этом отрезке.

Свойства неопределенного интеграла

1. Производная неопределенного интеграла равна подынтегральной функции; дифференциал неопределенного интеграла равен подынтегральному выражению:

2. Неопределенный интеграл от дифференциала функции равен этой функции, сложенной с произвольной постоянной, т. е.

.

3. Постоянный множитель можно   выносить за знак неопределенного интеграла:

4. Неопределенный интеграл от алгебраической суммы функций равен такой же алгебраической сумме неопределенных интегралов от каждой функции;

Основные формулы интегрирования (табличные интегралы).

Из каждой формулы дифференцирования вытекает соответствующая ей формула интегрирования. Например, из того, что , следует равенство

Ниже приведена таблица основных интегралов:

1. .

2. .

3. .

4. .

5. .

6. .

7. .

8. .

9. .

10. .

11. .

Справедливость этих формул можно проверить дифференцированием.

Непосредственное интегрирование. Под непосредственным интегрированием понимают такой способ интегрирования, при котором данный интеграл путем тождественных преобразований подынтегральной функции и применения свойств неопределенного интеграла приводится к одному или нескольким табличным интегралам.

Пример 1. Найти интеграл  

Решение Воспользуемся определением степени с отрицательным показателем (а-п=1/а", а0) и найдем неопределенный интеграл от степени:

Пример 2. Найти интеграл

Решение. Воспользуемся определением степени с дробным показателем () и найдем неопределенный интеграл от степени:

Пример 3. Найти интеграл

Решение. Воспользуемся определением степени с дробным и отрицательным показателем и правилом умножения степеней с одинаковыми основаниями (аmn = аm+n) и найдем неопределенный интеграл от степени:

Пример 4. Найти интеграл

Решение. Воспользуемся  определением  степени с дробным показателем     (),   правилами действий над степенями с одинаковыми основаниями (аmn = аm+n, am/an=am-n), правилом деления суммы на число и найдем интеграл от каждого слагаемого отдельно. Имеем

Заметим, что произвольные постоянные, входящие по определению в каждый из слагаемых неопределенных интегралов, объединяются в одну произвольную постоянную

Пример 5. Найти интеграл

Решение. Раскроем скобки по формуле (а—b)2=a2—2аb+b2 и неопределенный интеграл от полученной алгебраической суммы функций заменим такой же алгебраической суммой неопределенных интегралов от каждой функции:

Пример 6. Найти интеграл

Решение. Для нахождения интеграла воспользуемся формулой и свойствами неопределенного интеграла:

Интегрирование методом подстановки. Если интеграл затруднительно привести к табличному с помощью элементарных преобразований, то в этом случае пользуются методом подстановки.

Сущность этого метода заключается в том, что путем введения новой переменной удается свести данный интеграл к новому интегралу, который сравнительно легко берется непосредственно.

Для интегрирования методом подстановки можно использовать следующую схему:

1) часть подынтегральной функции надо заменить новой переменной;

2) найти дифференциал от обеих частей замены;

3) все подынтегральное выражение выразить через новую переменную (после чего должен получиться табличный интеграл);

4) найти полученный табличный интеграл;

5) сделать обратную замену.

Пример 7. Найти интеграл  

Решение. Произведем подстановку 5 — Зх = t, тогда — 3dх=dt, откуда dx= . Далее получаем

Пример 8. Найти интеграл

Решение. Сначала положим 2+соs x=t, тогда -sin хdх=dt, откуда sin хdх=—dt. Далее получаем

Пример 9. Найти интеграл  

Решение. Положим 2+Зеx = t  откуда . Далее получаем

Пример 10. Найти интеграл  .

Решение. Положим x/2=t, тогда . Далее получаем

В практике интегрирования часто встречаются интегралы, для нахождения которых можно использовать следующие формулы (k0, n0 — постоянные) :

; ;

; ; ;

;

Так, при нахождении можно использовать формулу , где k=1/2. Тогда  

Вопросы и упражнения для самопроверки

1. Какое действие называется интегрированием?

2. Какая функция называется первообразной для функции f (х) ?

3. Дайте определение неопределенного интеграла.

4. Перечислите основные свойства неопределенного интеграла.

5. Каким действием можно проверить интегрирование?

6. Напишите основные формулы интегрирования (табличные интегралы).

7. Найдите интегралы:

а) ; б) ; в) ; г) ; д) ;

е) ; ж) ; з) .

Ответы. 7. а) ; б) ; в) ; г) ;

д) ; е) ; ж) ; з) .

Определенный интеграл

По данной теме сначала изучите § 7—10 (1, 2), 11 '(1, 2) гл. 3, § 12, 14 (1—3) гл. 4 [3] или § 1—14 гл. 8 [7]. Затем ознакомьтесь с методическими указаниями по этой теме и внимательно разберите решение примеров из данного пособия. Ответьте на вопросы и выполните упражнения для самопроверки. Решите следующие задачи: [3]. гл. 3, § 10, КЬ 3.5—3.8, 3.12; гл. 4, § 12, № 4.1, § 14, № 4.3—4.29 или [7], гл. 8, № 1—5, 8—13, 17—21, 23—27, 42—49, 50—55, 60—63.

7—347

Из контрольной работы 2выполните второе и третье задания своего варианта.

Понятие определенного интеграла. Пусть функция f(х) определена на отрезке аb. Допустим для простоты, что функция f(х) в указанном промежутке неотрицательна и а<b. Разобьем этот отрезок на n частей точками а=x0<x12<...<хn=b. На каждом из частичных отрезков xi-1xi (i=1, 2, 3, ..., n) возьмем произвольную точку сi и составим сумму:

где . Эта сумма носит название интегральной суммы функции f(х) на отрезке аb.

Геометрически (рис. 52) каждое слагаемое интегральной суммы равно площади прямоугольника с основанием и высотой fi), а вся сумма равна площади «ступенчатой фигуры», получающейся объединением всех указанных выше прямоугольников.

Очевидно, что при всевозможных разбиениях отрезка аb.на  части получим различные интегральные суммы, а следовательно, и различные «ступенчатые фигуры».

Будем увеличивать число точек разбиения так, чтобы длина наибольшего из отрезков xi-1xi стремилась к нулю. Во многих случаях при таком разбиении интегральная сумма будет стремиться к некоторому конечному пределу, не зависящему ни от способа, каким выбираются точки деления хi ни от того, как выбираются промежуточные точки сi.

Этот предел и называется определенным интегралом от функции f(x) на отрезке аb.

Определенным интегралом от функции f(х) на отрезке аb.называется предел, к которому стремится интегральная сумма при стремлении к нулю длины наибольшего частичного интервала. Он обозначается символом и читается «интеграл от а до b от функции f(х) по dх» или, короче, «интеграл от а до b от f(x) по dx». По определению,

Число а называется нижним пределом интегрирования, число b — верхним; отрезок аb.— отрезком интегрирования.

Заметим, что всякая непрерывная на отрезке а≤xb функция f(х) интегрируема на этом отрезке.

Если интегрируемая на отрезке аb.функция f(х) неотрицательна, то определенный интеграл численно равен площади s криволинейной трапеции аАВb, ограниченной графиком функции у = f(х), осью абсцисс и прямыми х = а и х = b (рис. 52), т. е. s = В этом заключается геометрический смысл определенного интеграла.

Основные свойства определенного интеграла. Все свойства сформулированы в предположении, что рассматриваемые функции интегрируемы в соответствующих промежутках.

  1. Определенный интеграл с одинаковыми пределами равен нулю:

  1. При перестановке пределов интегрирования  знак интеграла меняется на противоположный:

  1. Отрезок интегрирования можно разбивать на части:

, где a < c < b.

  1. Постоянный множитель можно выносить за знак интеграла:

  1. Интеграл от алгебраической суммы функций равен такой же алгебраической сумме интегралов от всех слагаемых:

Непосредственное вычисление определенного интеграла. Для вычисления определенного интеграла, когда можно найти соответствующий неопределенный интеграл, служит формула Ньютона — Лейбница

т. е. определенный интеграл равен разности значении любой первообразной функции при верхнем и нижнем пределах интегрирования.

Из этой формулы виден порядок вычисления определенного интеграла:

1) найти неопределенный интеграл от данной функции;

2) в полученную первообразную подставить вместо аргумента сначала верхний, затем нижний предел интеграла;

3) из результата подстановки верхнего предела вычесть результат подстановки нижнего предела.

Пример 1. Вычислить интеграл

Решение. Применив указанное правило, вычислим данный определенный интеграл:

.

Пример 2. Вычислить интеграл

Решение. Воспользуемся определением степени с дробным и отрицательным показателем и вычислим определенный интеграл:

.

Пример 3. Вычислить интеграл

Решение. Интеграл от разности функций заменим разностью интегралов от каждой функции:

.

Пример 4. Вычислить интеграл

Решение. Воспользуемся определением степени с дробным показателем, правилом деления суммы на число и вычислим определенный интеграл от каждого слагаемого отдельно:

.

Вычисление определенного интеграла методом подстановки. Вычисление определенного интеграла методом подстановки состоит в следующем:

1) часть подынтегральной функции заменить новой переменной;

2) найти новые пределы определенного интеграла;

3) найти дифференциал от обеих частей замены;

4) все подынтегральное выражение выразить через новую переменную (после чего должен получиться табличный интеграл);

5) вычислить полученный определенный интеграл.

Пример 5. Вычислить интеграл

Решение. Введем подстановку 8 — х = t, тогда –dx=dt, dx=-dt. Определим пределы интегрирования для переменной t. При х=0 получаем tн = 8 — 0 = 8, при х=7 получаем tВ = =8 — 7 = 1.

Выразив подынтегральное выражение через t и dt и перейдя к новым пределам, получим

Пример 6. Вычислить интеграл

Решение. Произведем подстановку x3+2=t, тогда 3x2dx=dt, x2dx=. Определим пределы интегрирования для переменной t. При х = 1 получаем tн=13+ 4-2=3, при x: = 2 получаем tв==23+2=10.

Выразив подынтегральное выражение через t и dt а перейдя к новым пределам, получим

.

Пример 7. Вычислить интеграл   

Решение. Положим cosx=t, тогда –sin х dx=dt  и sin х dx = — dt. Определим пределы интегрирования для переменной t : tн = cos0=1, tв = соs(/2) =0.

Выразив подынтегральное выражение через t и dt и перейдя к новым пределам, получим

Пример 8. Вычислить интеграл  

Решение.

Пример 9. Вычислить интеграл

Решение. Сначала преобразуем подынтегральное выражение:

Затем вычислим интеграл от разности функций, заменив его разностью определенных интегралов от каждой функции:

.

Вычислим каждый интеграл отдельно:

Приложения определенного интеграла. Понятие определенного интеграла широко применяется для вычисления различных геометрических и физических величин.

Площади плоских фигур. Площадь криволинейной трапеции аАВb (рис. 53), ограниченной графиком непрерывной функции у = f(х) (где аb), отрезком ab оси Оx: и отрезками прямых х=а и х = b, вычисляется по формуле

, где  (1)

Пример 10. Вычислить площадь фигуры, ограниченной гиперболой ху=1, осью Ох и прямыми x = 1 и х  = е (рис. 54).

Решение. Применяя формулу (1), получаем

, кв. ед.

Пример 11. Вычислить площадь фигуры, ограничен.ной параболой у = х2, прямыми х = -1,  х=2 и осью абсцисс  (рис. 55).

Решение. Применяя формулу (1), получаем

кв. ед.

Площадь фигуры ABCD {рис. 56), ограниченной графиками непрерывных функций у=f1(х) и у=f2(x) (где аb) и отрезками прямых х = а и х=b, вычисляем по формуле , где  (2).

Пример 12. Вычислить площадь фигуры, ограниченной кривой у=6х — х2 — 5 и осью Оx (рис. 57).

Решение. Найдем пределы интегрирования, т. е, абсциссы точек пересечения графиков функций у=6х — х2 — 5 и у=0 (ось Ох). Для этого решим систему

Имеем 6х— х2— 5=0,   x2-6x+5=0,   x1,2=,  x1=1, x2=5.

Теперь найдем искомую площадь по формуле (2):

кв. ед.

 Пример 13. Вычислить площадь фигуры, ограниченной линиями у=х2 и у2=х (рис. 58).

Решение. Найдем пределы интегрирования, т. е. абсциссы точек пересечения графиков функций у=х2 и у2=х. Для этого решим систему

Имеем (х2)2=х, х4—х=0, х(х3—1)=0, x1=0, x2=1. Искомую площадь вычисляем по формуле (2) при f1(x)=x2, f2(x)=

,

кв. ед.

Пример 14. Вычислить площадь фигуры, ограниченной параболами у = 4 — х2 и у=х2 —2х (рис. 59).

Решение. Найдем пределы интегрирования, т. е. абсциссы точек пересечения графиков функций y=4 - x2 и у=х2 — 2х. Для этого решим систему

Имеем 4 – x2 = x2 – 2x, 2x2 - 2x – 4 = 0, x2x – 2 = 0

Искомую площадь вычисляем по формуле (2):

Объем тела вращения. Объем тела, образованного вращением вокруг оси Ох криволинейной трапеции аAВb, ограниченной непрерывной кривой у=f(х) (где аb ), отрезком аb оси Ох и отрезками прямых х = а и х=b (рис. 60), вычисляется по формуле

 (3)

Пример 15. Вычислить объем тела, образованного вращением вокруг оси Ох фигуры, ограниченной параболой у2 = 2x, прямой х=3 и осью Ох (рис. 61).

Решение. Применяя формулу (3), находим

;   куб. ед.

Пример 16. Вычислить объем шара радиуса К (рис. 62).

Решение. Шар образован вращением вокруг оси Ох круга, ограниченного окружностью х22 = R2 с центром в начале координат и радиусом R. Учитывая симметрию

круга относительно оси ординат, сначала найдем по формуле (3) половину искомого объема:

 

куб. ед. Следовательно, куб.ед.

Пример 17. Вычислить объем тела, образованного вращением вокруг оси Ох фигуры, ограниченной осью Ох и полуволной синусоиды у = sin x (0) (рнс. 63).

Решение. Применяя формулу (3), находим

;

куб. ед.

Объем тела, образованного вращением вокруг оси Оу криволинейной трапеции аАВb ограниченной непрерывной кривой х = f(у) (где аb), отрезком аb оси Оу и отрезками пряных у = а и у=b (рис. 64), вычисляется по формуле

/

Пример 18. Вычислить объем тела, образованного вращением вокруг оси Оу фигуры, ограниченной параболой у=х2 и прямой у=4 (рис. 65).

Решение. Применяя формулу (4), находим

куб. ед.

Пример 19. Вычислить объем тела, образованного вращением вокруг оси Оу фигуры, ограниченной параболой у=х2+1 и прямой у = 2 (рис. 66).

Решение. Объем полученного тела  (оно называется параболоидом) вычислим по формуле (4);

; куб.ед.

Путь, пройденный точкой. Если точка движется прямолинейно и ее скорость v=f(t) есть известная функция времени t, то путь, пройденный точкой за промежуток времени вычисляется по формуле

 (5)

Пример 20. Тело движется прямолинейно со скоростью v=0.1t3(v—в м/с), Вычислить путь, пройденный телом за 10 с.

Решение. Применяя формулу (5), находим

(м).

Пример 21. Скорость прямолинейно движущегося тела равна v=(4tt2) (v — в м/с). Вычислить путь, пройденный телом от начала движения до остановки.

Решение. В момент остановки скорость движения тела равна нулю, т. е. 4tt2 = 0, t(4-t)=0, t2=4.

Итак, тело остановится через 4 с.

Путь, пройденный телом за это время, вычисляем по формуле (5):

(м).

Работа силы. Если переменная сила F=F(х) действует в направлении оси Ох, то работа силы на отрезке вычисляется по формуле

 (6)

Пример 22. Вычислить работу, которую нужно совершить при сжатии пружины на 0,08 м, если для ее сжатия на 1 см требуется сила 10 Н.

Решение. Согласно закону Гука, сила F, растягивающая или сжимающая пружину на x метров, равна F=kх, где k — коэффициент пропорциональности.

Из условия следует 10 = k*0,01  т. е. k = 1000, и следовательно, F=kx=1000x

Искомую работу находим по формуле (6) :

(Дж).

Пример 23. Сила 196,2 Н растягивает пружину на 18 см. Какую работу она производит?

Решение. По закону Гука, F=kх, откуда k=F/х = 196,2/0,18=1090 (Н/м). Значит, k=1090 х. Находим искомую работу:

(Дж).

Пример 24. Для сжатия пружины на 3 см необходимо совершить работу в 16 Дж. На какую длину можно сжать пружину, совершив работу в 144 Дж?

Решение. По закону Гука, F = kх; тогда

.

Так как Дж, то , откуда .

Значит, . Далее, имеем

.

Но Дж, т.е.

Итак, пружину можно сжать на 9 см.

Сила давления жидкости. Сила давления р жидкости плотности р на вертикальную пластинку, погруженную в жидкость, вычисляется по формуле

 (7)

где g = 9,81 м/с2 — ускорение свободного падения, s — площадь пластинки, а глубина погружения пластинки изменяется от а до b.

Пример 25. Вычислить силу давления воды на одну из стенок аквариума, имеющего длину 30 см и высоту 20 см.

Решение. Стенка аквариума имеет форму прямоугольника, поэтому s = 0,3 х, где 00,2. Плотность воды равна 1000 кг/м3. Тогда сила давления воды на стенку аквариума согласно формуле (7) составляет

(Н).

Пример 26. Вычислить силу давления бензина на стенки цилиндрического бака высотой 3 м и радиусом основания 1 м.

Решение. Площадь поверхности стенки цилиндрического бака s=2 = 2, где 03. Плотность бензина есть 800 кг/м3. Тогда сила давления бензина на стенки бака составляет

(Н).

Пример 27. Вычислить силу давления воды на погруженную в нес вертикальную пластинку, имеющую форму треугольника с основанием 6 м и высотой 2 м, предполагая, что вершина этого треугольника лежит на свободной поверхности воды, а основание параллельно ей (рис.67).

Решение. Пусть МN — ширина пластинки на уровне ВЕ=х. Из подобия треугольников АВС и МВN находим

, или

На основании формулы (7) получаем

(Н)

Вопросы и упражнения для самопроверки

1. Дайте определение определенного интеграла.

2. Перечислите основные свойства определенного интеграла.

3.-В чем заключается геометрический смысл определенного интеграла?

4. Напишите формулы для определения площади плоской фигуры с помощью определенного интеграла.

5. По каким формулам находится объем тела вращения?

6. Напишите формулу для вычисления пути, пройденного телом.

7. Напишите формулу для вычисления работы переменной силы.

8. По какой формуле вычисляется сила давления жидкости на пластинку?

9. Вычислите определенные интегралы: а) ; б) ; в) ; г) .

10. Вычислите площадь фигуры, ограниченной линиями: а) у=х2+.1, у=0, х=—2, х = 1, б) х2—9у = 0 и x-3y+6=0

11. Вычислите объем тела, образованного вращением вокруг оси Ох фигуры, ограниченной параболой у2=х, прямой х = 2 и осью Ох.

12. Вычислите объем тела, образованного вращением вокруг оси Ох фигуры, ограниченной параболой У2=x, прямыми у=1, y = 4 и осью Оу.

13. Тело движется прямолинейно со скоростью v = (2 + 4t3) (v — в м/с). Вычислите путь, пройденный телом за первые три секунды.

14. Какую работу нужно совершить, чтобы растянуть пружину на 6 см, если сила в 1 Н растягивает ее на 1 см?

15. Вычислите силу давления воды на вертикальный прямоугольный шлюз с основанием 18 м и высотой 6 м.

Ответы. 9. а) 19; б) 4е; в) 8/3: г) 2/9. 10.а) 6 кв.ед.; б) 13,5 кв. ед„ 11. 2л куб. ед. 12. 12п куб. ед. 13. 87 м. 14. 0,18 Дж. 15. 3178440 Н.

Дифференциальные уравнения

По данной теме сначала изучите § 24—30 гл. 8 [3] или § 1, 7 гл. 10 [7]. Затем ознакомьтесь с методическими указаниями по этой теме и внимательно разберите решение примеров из данного пособия. Ответьте на вопросы и выполните упражнения для самопроверки. Решите следующие задачи: [3], гл. 8, § 24, № 8.1—8.35 или [7], гл. 10, № 1—89.

Из контрольной работы 2 выполните четвертое задание своего варианта.

Понятие о дифференциальном уравнении. Дифференциальным уравнением называется уравнение, связывающее независимую переменную, искомую функцию, ее производную (или дифференциал аргумента и дифференциал функции).

Если дифференциальное уравнение содержит производную или дифференциал не выше первого порядка, то оно называется дифференциальным уравнением первого порядка. Общий вид такого уравнения F(х, у, у') =0, где у`=f`(х)—искомая неизвестная функция у'=f'(Х)— ее производная по х, а F — заданная функция переменных х, у, у'.

Общим решением дифференциального уравнения первого порядка называется функция у=ф(х, с) от х и произвольной постоянной С, обращающая это уравнение в тождество по х.

Общее решение, записанное в неявном виде Ф(х, у, С) =0, называется общим интегралом.

Частным решением уравнения F(х, у, у') =0 называется решение, полученное из общего решения при фиксированном значении C: у=φ(х, С0), где С0 — фиксированное число.

Частным интегралом уравнения F(х, у, у')=0 называется интеграл, полученный из общего интеграла при фиксированном значении С: Ф(х, у, С0) =0,

График любого частного решения дифференциального уравнения F(х, у, у')=0 называется интегральной кривой. Общему решению (и общему интегралу) этого уравнения соответствует семейство интегральных кривых, зависящих от одного параметра.

Задача нахождения частного интеграла дифференциального уравнения n-го порядка .(n = 1, 2, 3, ...), удовлетворяющего начальным условиям вида у(х) =уо, у'(х0)=y`0 . называется задачей Коши.

Задача Коши для дифференциального уравнения первого порядка состоит в том, чтобы найти решение, удовлетворяющее начальному условию у(хо)=уо, Другими словами, из всех интегральных кривых данного дифференциального уравнения требуется выделить ту, которая проходит через данную точку (x0, y0)

Пример 1. Составить уравнение кривой у = f(х), если угловой коэффициент касательной, проведенной в любой точке кривой, равен 2х.

Решение. Так как на основании геометрического смысла производной y` = kкас , то получим дифференциальное уравнение первого порядка:

, ,

Чтобы найти искомую функцию у=f(х), надо проинтегрировать обе части уравнения . Отсюда получим общее решение дифференциального уравнения:. Геометрически это решение представляет собой семейство парабол с вершиной на оси Оу, симметричных относительно этой оси (рис. 68).

Чтобы из общего решения выделить частное решение, надо задать начальные условия, Пусть у=-1 при х = 1; тогда общее решение примет вид —1 = 1 + С, откуда С=—2. Геометрически частное решение у=х2—2 представляет собой параболу, проходящую через точку.(1, —1) (рис.68).

Дифференциальные уравнения с разделяющимися переменными. Общий вид такого уравнения

,

где Х(х), Х1(х) — функции только от х, У(у), У1 (у) — функции только от у

Поделив обе части уравнения на произведение Х1(х)У(у)0, получим уравнение с разделенными переменными

.

Общий интеграл этого уравнения имеет вид

.

Замечание. Если произведение Х1 (х) У(у) =0 при х=а и у=b, то эти функции х=а и у=b являются решениями дифференциального уравнения при условии, что при этих значениях х и у уравнение не теряет числового смысла. Геометрически эти решения представляют собой прямые, параллельные осям координат.

Пример 2. Решить уравнение y dy = x dx. Найти частное решение, удовлетворяющее условию y=4 при х = —2.

Решение. Это уравнение с разделенными переменными. Интегрируя, находим общее решение уравнения:

.

Для получения более простого по форме общего решения постоянное слагаемое в правой части представлено в виде С/2. Тогда y2=x2+c

Подставив в общее решение значения y=4 и х= — 2, получим 16 = 4+С, откуда С=12.

Итак, частное решение уравнения, удовлетворяющее данному условию, имеет вид y2=x2+12.

Пример 3. Найти общий интеграл дифференциального уравнения ху' — у=y3

Решение. Так как у' =

, откуда xdy=(y3+y)dx

Разделим обе части уравнения на произведение xy(y2+1)

. Преобразуем дробь:

Тогда . Интегрируя, находим ,

,

.

Для облегчения потенцирования и получения более простого по форме общего решения постоянное слагаемое в правой части представлено в виде ln . После потенцирования получим

откуда или где .

Произведение ху(у2+1)=0 при x = 0 и при у = 0. При этих значениях х и у дифференциальное уравнение не теряет числового смысла, поэтому x = 0 и y = 0 — решения уравнения, но решение у = 0 входит в решение

= Cx при С=0

Значит, решения уравнения имеют вид =Сx и x=0

Пример 4. Решить уравнение 2x siny dx + (x2+3).cosy dy=0 Найти частное решение, удовлетворяющее условию y==π/2 при х=1.

Решение. Разделим каждый член уравнения на произведение (x2+3) sin y:

Интегрируя, находим , ,

.

После потенциирования получим , или , где . Отсюда .

Произведение при sin y = 0; так как при этом значении дифференциальное уравнение не теряет числового смысла, то sin y = 0 – решение уравнения. Но оно входит в интеграл при С = 0. Значит, общий интеграл уравнения имеет вид .

Подставив в общий интеграл значения и х = 1, получим 1 = С/ 4, откуда С=4. Итак, частный интеграл уравнения, удовлетворяющий данному условию, имеет вид .

Пример 5. Решить уравнение . Найти частное решение, удовлетворяющее условию у = 1 при х = 0.

Решение. Так как , то , откуда .

Разделим обе части уравнения на : .

Интегрируя, находим , или .

После потенциирования получим решение .

При у = 1 и х = 0 имеем , , откуда .

Итак, частный интеграл уравнения, удовлетворяющий данному условию, имеет вид

или .

Пример 6. Решить уравнение . Найти частное решение, удовлетворяющее условию у = 3 при .

Решение. Так как , то , откуда .

Разделим обе части уравнения на произведение :

.

Интегрируя, находим:

, .

После потенциирования получим решение , откуда или , где .

Произведение при у = 0; так как при этом значении у дифференциальное уравнение не теряет числового смысла, то у = 0 – решение уравнения. Но оно входит в решение при С = 0. Значит, общее решение уравнения имеет вид: .

Подставив в общее решение значение у = 3 и , получим 3 = С*3, откуда С = 1. Итак, частное решение уравнения, удовлетворяющее данному условию, имеет вид .

Линейные дифференциальные уравнения первого порядка. Общий вид такого уравнения

,  (1)

где f(x) и q(x) – заданные функции от х. Это уравнение является линейным относительно искомой функции и ее производной.

Есди q(x)=0, то линейное дифференциальное уравнение (1) называется однородным. Оно имеет вид и решается методом разделения переменных:

, ,

, ,

,

где F(x) – некоторая первообразная функции f(x), а - произвольная постоянная.

Еслн f(x)=0, то уравнение (1) принимает вид у'=q(x) и решается методом разделения переменных:

 ,

где Q(x)—некоторая первообразная функции q(x), а С — произвольная постоянная.

Существуют различные приемы решения линейного неоднородного дифференциального уравнения. Рассмотрим два из них.

  1. Этот прием решения основан на применении следующей теоремы: если у=φ(х)—-некоторое решение уравнения (1), то все решения этого уравнения задаются формулой

,

где СеF(x) — общее решение однородного уравнения. Иными словами, для нахождения общего решения уравнения (1) достаточно найти хотя бы одно его частное решение,

Пример 7. Найти общее решение дифференциального уравнения y`+y=x+1.

Решение. Подбором находим, что функция у=х является решением данного линейного неоднородного уравнения. Найдем общее решение соответствующего однородного уравнения;

где

Общее решение данного уравнения имеет вид у = Се-x +x

2. Этот прием решения основан на простом замечании, что любую величину t (переменную или постоянную) можно представить в виде произведения двух множителей: t = uv, причем один из них можно выбрать произвольно (лишь бы он был отличен от нуля).

Например, в равенстве tgх=uv можно взять v = еx, тогда и = е-xtg х можно взять , тогда u= и т. д.

Решение линейного неоднородного дифференциального уравнения будем искать в виде у = uv, где u и v — функции от х.

Пример 8. Найти общее решение дифференциального уравнения

Решение. Данное уравнение является линейным. Полагаем у = uv, тогда у' = u`v+v`u и уравнение преобразуется к виду

или

Так как один множитель можно выбрать произвольно, то выберем в качестве v какой-либо частный ннтеграл уравнения v'—v сtg x: = 0. Тогда для отыскания и получим уравнение u`v=sinx

Решим уравнение v'—v сtg x: = 0.; имеем

, , , , v = sinx.

В качестве v выбран частный интеграл уравнения при С = 0.

Подставляя значение v во второе уравнение и решая его, найдем и, как общий интеграл этого уравнения:

u` sin x=sin x, u`=1, du=dx, u-x+c

Зная u и v, находим искомую функцию у:

y=(x+c)sin x.

Пример 9. Найти частное решение дифференциального уравнения , удовлетворяющее условию у=0 при х: = 0.

Решение. Положим y = uv; тогда у'= u'v+v`u и данное уравнение примет вид

или .

Положим , тогда  

Проинтегрировав, получим частное решение lnv=-x2  или .  При уравнение примет вид

, , , откуда .

Общее решение данного дифференциального уравнения:

.

Подставив в это равенство начальные условия, получим 0 = е°(0+С), откуда С=0.

Итак, частное решение данного дифференциального уравнения, удовлетворяющее данному условию, имеет вид .

Дифференциальные уравнения второго порядка. Если дифференциальное уравнение содержит производную или дифференциал не выше второго порядка, то оно называется дифференциальным уравнением второго порядка.

Общий вид такого уравнения ,

где у=f(х)—искомая неизвестная функция, у'=f'(х) и у"=f”(х)—ее производные по х первого и второго порядков, а F — заданная функция переменных х, у, у', y".

Общим решением дифференциального уравнения второго порядка называется функция y=φ(x, C1, C2) от х и двух произвольных постоянных С1 и С2, обращающая это уравнение в тождество по х.

Общее решение, записанное в неявном виде Ф(х, у, С1 С2) =0, называется общим интегралом.

Частным решением уравнения F(х, у, у', у")=0 называется решение, полученное из общего решения при фиксированном значении С1 и С2: у=φ(х, С10, С20 ), где С10 и С20 — фиксированные числа.

Частным интегралом этого уравнения называется интеграл, полученный из общего интеграла при фиксированном значении С1 и С2: Ф(х, у, С10, С20)=0, где С10 и С20 — фиксированные числа.

Общее решение дифференциального уравнения F(х, y, у', у"}=0 можно рассматривать как семейство интегральных кривых данного уравнения, зависящее от двух параметров С1 и С2. Частному решению, полученному из общего, соответствует одна кривая этого семейства.

Задача Коши для дифференциального уравнения второго порядка состоит в том, чтобы найти решение, удовлетворяющее начальным условиям у(хо)=уо, у'(хо)=у'0. Постоянные С1 и С2 определяются из системы уравнений

Другими словами, из всех интегральных кривых данного дифференциального уравнения требуется выделить интегральную кривую, проходящую через данную точку (х0, уо) в заданном направлении у' (Хо).

Линейные однородные дифференциальные уравнения второго порядка с постоянными коэффициентами. Линейным дифференциальным уравнением второго порядка с постоянными коэффициентами называется уравнение вида

.

где р и q — некоторые числа.

Если f(х)=0, то дифференциальное уравнение называется линейным однородным. Оно имеет вид

(2)

Справедлива теорема: если у1 и у2 —частные решения уравнения (2), причем у1/у2const то функция У=С1у1+С2y2, где С1 и С2 —произвольные постоянные, является общим решением этого уравнения.

Решением данного дифференциального уравнения (2) должна быть такая функция, которая, будучи подставлена в уравнение, превратит его в тождество. Левая часть уравнения представляет собой сумму функции у и ее производных у' и у", взятых с некоторыми постоянными коэффициентами. Чтобы такая сумма обратилась в нуль, надо, чтобы y, у' и у" были подобны между собой.

Такой функцией является функция у = еkx, где k — постоянная. Требуется подобрать k так, чтобы эта функция удовлетворяла уравнению (2).

Так как у' = еkx (kх)' = k еkx , а y" =k еkx (kx:)'=k2 еkx , то, подставляя эти значения у, у' и у" в левую часть уравнения (2), получим

.

Сокращая на множитель еkx, не обращающийся в нуль, получим характеристическое уравнение

 (3)

Это уравнение определяет те значения А, при которых функция у = еkx является решением дифференциального уравнения (2).

При решении характеристического уравнения (3) возможны три случая:

Корни уравнения

Частные решения

Общее решение

1

Действительные различные (k1k2)

Y1=ek1x

Y2=ek2x

Y=C1ek1x+C2 ek2x

2

Действительные равные (k1=k2)

Y1=ek1x

Y2=xek1x

Y=ek1x(C1+C2x)

3

Комплексно-сопряженные ()

Y1=eaxcos, Y2=eaxsin

Y=eax(cos+C2 sin)

Пример 10. Найти общее решение дифференциального уравнения

.

Решение. Составим характеристическое уравнение и найдем его корни:

.

Корни характеристического уравнения являются действительными и различными. Поэтому у12x, y2 = е3x— частные решения, а у = С1e+-С2е — общее решение данного дифференциального уравнения.

Пример 11. Найти общее решение дифференциального уравнения .

Решение. Характеристическое уравнение k2+4k+4=0 или {k+2)2 = 0 имеет действительные равные корни k1=k2=-2, Поэтому y1= е-2х, у2=хе-2х — частные решения, а =е-2х12х) — общее решение данного дифференциального уравнения.

Пример 12. Найти общее решение дифференциального уравнения .

Решение. Составим характеристическое уравнение и найдем его корни:

;

Корни уравнения являются комплексно-сопряженными. Поэтому у1-3xсоs 2х, у2 = е-3x sin 2х — частные решения, а у = е-3x1 cos2x+C2sin2x) — общее решение данного дифференциального уравнения.

Пример 13. Найти частное решение дифференциального уравнения у"—2y'+y = 0, удовлетворяющее начальным условиям при x=0, y=4, у' = 2-

Решение. Характеристическое уравнение k2+2k+1=0 или (k-1)2 = 0 имеет действительные равные корни k1=k2=-2, поэтому y1=ex, y2=xex — частные решения, а у=ех12х) —общее решение данного дифференциального уравнения.

Для определения частного решения, удовлетворяющего данным начальным условиям, сначала найдем производную у' функции у = ех (12х):

.

Теперь подставим начальные условия  в выражения для у и у':

или ,

откуда С1=4, С2=-2

Подставив эти значения в общее решение, найдем частное решение дифференциального уравнения, удовлетворяющее данным начальным условиям: у = еx(4 —2х).

Вопросы и упражнения для самопроверки

1. Какое уравнение называется дифференциальным?

2. Дайте определение дифференциального уравнения первого порядка.

3. Дайте определение общего решения и общего интеграла дифференциального уравнения первого порядка.

4. Дайте определение частного решения и частного интеграла дифференциального уравнения первого порядка.

5. Решите дифференциальные уравнения и найдите частные решения (частные интегралы), удовлетворяющие данным условиям:

а) при х=2; б) при х = 0;

в) при х = 0;

г) при х = 1.

Ответы. 5. а) ; б) , ;

в) ; г) , .

Элементы теории вероятностей

По данной теме сначала изучите § 18 (1—3) гл. 6, § 20—23 гл. 7 13] или § 1—9 гл. II [7]. Затем ознакомьтесь с методическими указаниями по этой теме и внимательно разберите решение примеров из данного пособия. Ответьте на вопросы и выполните упражнения для самопроверки. Решите следующие задачи: [3], гл. 6, § 18, № 6.1—6.11; гл. 7, § 20, №7.1—7.8 или [7], гл. 11, ЛЬ 1—16.

Из контрольной работы 2 выполните шестое задание своего варианта.

Основные понятия комбинаторики. Задачи, при решении которых приходится составлять различные комбинации из конечного числа элементов и производить подсчет числа всех возможных таких комбинаций, называются комбинаторными.

Этот раздел математики находит широкое практическое применение во многих вопросах естествознания и техники.

Размещения. Пусть имеется множество, содержащее n элементов. Каждое его упорядоченное подмножество, содержащее по m элементов, называется размещением из n элементов по m элементов.

Из определения вытекает, что и что размещения из n элементов по m — это все m-элементные подмножества, отличающиеся составом элементов или порядком их следования,

Число размещений из n элементов по m элементов в каждом обозначают и вычисляют по формуле

.

Число размещений из n элементов по m элементов в каждом равно произведению m последовательно убывающих натуральных чисел, из которых большее есть n.

Для кратности произведение первых n натуральных чисел принято обозначать n! (n-факториал):

1•2•3 •…•  n = n!

Условились считать, что 0! = 1.

Тогда формулу числа размещений из n элементов по m элементов можно записать в другом виде:

/

Условились считать, что = 1.

Пример 1. Сколькими способами из группы, включающей 25 учащихся, можно выбрать актив группы в составе старосты, комсорга и профорга?

Решение. Состав актива группы является упорядоченным множеством из 25 элементов по три элемента. Значит, искомое число способов равно числу размещений из 25 элементов по три элемента в каждом:

или .

Пример 2. Перед выпуском группа учащихся в 30 человек обменялась фотокарточками. Сколько всего было роздано фотокарточек?

Решение. Передача фотокарточки одним учащимся другому есть размещение из 30 элементов по два элемента. Искомое число фотокарточек равно числу размещений из 30 элементов по два элемента в каждом:

.

Перестановки. Размещения из n элементов по n элементов называются перестановками из n элементов.

Из определения следует, что перестановки являются частным случаем размещений. Так как каждая перестановка содержит все n элементов множества, то различные перестановки отличаются друг от друга только порядком элементов.

Число перестановок из n элементов данного множества обозначают Рn и вычисляют по формуле

Рn= 1•2•3 •…• n = n!.

Пример 3. Сколько четырехзначных чисел можно составить из цифр 1, 2, 3, 4 без повторений?

Решение. По условию дано множество из четырех элементов, которые требуется расположить в определенном порядке. Значит, требуется найти количество перестановок из четырех элементов:

Р4= 1•2•3•4 = 24,

т. е. из цифр 1, 2, 3, 4 можно составить 24 четырехзначных числа (без повторений цифр).

Пример 4. Сколькими способами можно рассадить 10 гостей по десяти местам за праздничным столом?

Решение. Искомое число способов равно числу перестановок из десяти элементов: Р10=10!=3628800

Сочетания. Пусть имеется множество, состоящее из n элементов. Каждое его подмножество, содержащее m элементов, называется сочетанием из n элементов по m элементов.

Таким образом, сочетания из n элементов по m элементов — это все m-элементные подмножества n-элементного множества, причем различными подмножествами считаются только те, которые имеют неодинаковый состав элементов.

Подмножества, отличающиеся друг от друга порядком следования элементов, не считаются различными.

Число подмножеств по m элементов в каждом, содержащихся во множестве из n элементов, т. е. число сочетаний из n элементов по т элементов в каждом, обозначают и вычисляют по формуле

или

Число сочетаний Собладает следующим свойством:

.

Так,

Пример 5. Сколько всего игр должны провести 20 футбольных команд в однокруговом чемпионате?

Решение. Так как игра любой команды А с командой В совпадает с игрой команды В с командой A, то каждая игра есть сочетание из 20 элементов по 2. Искомое число всех игр равно числу сочетаний из 20 элементов по 2 элемента в каждом:

.

Пример 6. Сколькими способами можно распределить 12 человек по бригадам, если в каждой бригаде по 6 человек?

Решение. Состав каждой бригады является конечным множеством из 12 элементов по 6. Значит, искомое число способов равно числу сочетаний из 12 элементов по 6 в каждом:

.

Случайные события. Вероятность события. Теория вероятностей — это математическая наука, которая изучает закономерности в случайных событиях. К основным понятиям теории вероятностей относятся испытания и события.

Под испытанием (опытом) понимают реализацию данного комплекса условий, в результате которого непременно произойдет какое-либо событие.

Например, бросание монеты — испытание; появление герба или цифры — события.

Случайным событием называется событие, связанное с данным испытанием, которое при осуществлении испытания может произойти, а может и не произойти. Слово «случайное» для краткости часто опускают и говорят просто «событие». Например, выстрел по цели — это опыт, случайные события в этом опыте — попадание в цель или промах,

Событие в данных условиях называется достоверным, если в результате опыта оно непременно должно произойти, и невозможным, если оно заведомо не произойдет. Например, выпадение не более шести очков при бросании одной игральной кости — достоверное событие; выпадение десяти очков при бросании одной игральной кости — невозможное событие,

События называются несовместными, если никакие два из них не могут появиться вместе. Например, попадание и промах при одном выстреле — это несовместные события.

Говорят, что несколько событий в данном опыте образуют полную систему событий, если в результате опыта непременно должно произойти хотя бы одно из них. Например, при бросании игральной кости события, состоящие в выпадении одного, двух, трех, четырех, пяти и шести очков, образуют полную систему событий.

События называются равновозможными, если ни одно из них не является объективно более возможным, чем другие. Например, при бросании монеты выпадение герба или числа — события равновозможные.

Каждое событие обладает какой-то степенью возможности. Числовая мера степени объективной возможности события — это вероятность события. Вероятность события А обозначается Р(А),

Пусть из системы n несовместных равновозможных исходов испытания m исходов благоприятствуют событию А. Тогда вероятностью события А называют отношение m числа исходов, благоприятствующих событию А, к числу всех исходов данного испытания:  P(A)=m/n

Эта формула носит название классического определения вероятности.

Если В — достоверное событие, то m = n и Р(В) = 1; если С — невозможное событие, то m= 0 и Р(С)=0; если А — случайное событие, то m и Р(А) 1.

Таким образом, вероятность события заключается в следующих пределах: 0Р(A).

Пример 7. Игральную кость подбрасывают один раз. Найти вероятность событий: А — появление четного числа очков; В — появление не менее пяти очков; С — появление не более пяти очков.

Решение. Опыт имеет шесть равновозможных независимых исходов (появление одного, двух, трех, четырех, пяти и шести очков), образующих полную систему.

Событию А благоприятствуют три исхода (выпадение двух, четырех и шести очков), поэтому Р(А}=3/6 = 1/2; событию В — два исхода (выпадение пяти и шести очков), поэтому Р(В)=2/6=1/3; событию С — пять исходов (выпадение одного, двух, трех, четырех и пяти очков), поэтому Р(С) =5/6.

При вычислении вероятности часто приходится использовать формулы комбинаторики, .

Рассмотрим примеры непосредственного вычисления вероятностей.

Пример 8. В урне находится 7 красных и 6 синих шаров. Из урны одновременно вынимают два шара. Какова вероятность того, что оба шара красные (событие А)?

Решение. Число равновозможных независимых исходов равно

.

Событию А благоприятствуют исходов. Следовательно, Р (А) =

Пример 9. В партии из 24 деталей пять бракованных. Из партии выбирают наугад 6 деталей. Найти вероятность того, что среди этих 6 деталей окажутся 2 бракованных (событие б).

Решение. Число равновозможных независимых исходов равно

.

Подсчитаем число исходов m, благоприятствующих событию В. Среди шести взятых наугад деталей должно быть 2 бракованных и 4 стандартных. Две бракованные детали из пяти можно выбрать способами, а 4 стандартных детали из 19 стандартных деталей можно выбрать способами. Каждая комбинация бракованных деталей может сочетаться с каждой комбинацией стандартных  деталей, поэтому m=3876*10=38760. Следовательно,

.

Пример 10. Девять различных книг расставлены наудачу на одной полке. Найти вероятность того, что четыре определенные книги окажутся поставленными рядом

(событие С).

Решение. Здесь число равновозможных независимых исходов есть n = Р9=9!. Подсчитаем число исходов m, благоприятствующих событию С. Представим себе, что четыре определенные книги связаны вместе, тогда эту связку можно расположить на полке

Р6 = 6! Способами (связка плюс остальные пять книг). Внутри связки четыре книги можно переставлять Р4 = 4! способами, При этом каждая комбинация внутри связки может сочетаться с каждым из Р6 способов образования связки,

т. е. m=6!*4!. Следователю, P (С)=

Вопросы  и упражнения для самопроверки

1. Какое событие называется невозможным; достоверным?

2. Какие события называются несовместными; равно возможными?

3. Какие события образуют полную систему событий?

4. Что понимается под вероятностью события?

5. Дайте классическое определение вероятности события.

6. а) В урне 3 белых и 9 черных шаров. Из урны наугад вынимают один шар. Какова вероятность того, что вынутый шар окажется черным?

б) В урне 4 красных и 7 синих шаров. Из урны одновременно вынимают два шара. Какова вероятность того, что оба шара красные?

в) Первенство по футболу оспаривают 18 команд, среди которых 5 лидирующих. Путем жеребьевки команды распределены на две группы по 9 команд в каждой. Какова вероятность попадания всех лидирующих команд в одну группу?

Ответы. 6. а) 3/4; б) 6/55; в) 1/3.




1. Минимизация и страхование проектных рисков в условиях рыночной экономики
2. Лебедь СенСанса
3.  Helthcre institutions in Kzkhstn
4. Психологические особенности пользователей сети Интерне
5. Анна Снегина справедливо считается одним из наиболее крупных по значению и масштабу творений Есенина произ
6. Число родившихся за период Р 2
7. докладе России на II Международном конгрессе ЮНЕСКО
8. Эволюция института местного самоуправления в Беларуси
9. Проблемы развития социально-экономической сферы России в 90-е годы
10. Аквариум
11. Статья- Оценка показателей физического развития и физической подготовленности юных квалифицированных тяжелоатлетов
12. Расторжение трудового договора по инициативе работодателя
13. Формирование техникоэкономических показателей производственной системы механосборочного производства.html
14. Особенности пещерного искусства История открытия и изучения (Ляско, Коске, Шове, Гаргас и другие)
15. Документационное обеспечение деятельности государственного учреждения
16. контрольна робота 1 з дисципліни Сучасна теорія управління Варіант 2 Виконав-
17. Курсовой проект Расчет материальных балансов процессов переработки танатарской нефти
18. це несвідоме Несвідоме являє собою сховище примітивних інстинктивних спонукань плюс емоції та спогади як
19. Христианские монастыри
20. темами Спеціальність 05.